You are on page 1of 57

Question Bank 6: FREQUENCY PROBLEM 3

MODULATION The operating frequency of an FM


transmitter is 168.96 MHz. Calculate the
PROBLEM 1 frequency of the carrier crystal oscillator if it
Using Carson’s Rule what is the bandwidth of uses three frequency multipliers – a doubler,
an FM signal with 5 kHz maximum frequency a tripler, and a quadrupler.
deviation and 2.5 kHz maximum modulating
frequency? Notations:
fo = oscillator operating frequency
Notations:
BW = Bandwidth Solution:
⸹max = maximum frequency deviation (Hz) The total multiplication produced by the
fm(max) = maximum modulating frequency frequency multipliers = 2 x 3 x 4 = 24.
(Hz) 168.96
𝑓𝑜 = = 7.04 𝑀𝐻𝑧
24
Solution: ⸫ The frequency of the carrier crystal
BW = 2(⸹max + fm(max)) oscillator is 7.04 MHz.
BW = 2(5 kHz + 2.5 kHz)
BW = 15 kHz PROBLEM 4
⸫ The bandwidth of FM signal is 15 kHz. What frequency deviation is caused by noise
in an FM receiver which has an input S/N of
PROBLEM 2 2.8 and the modulating frequency of 1.5
Calculate the maximum bandwidth required kHz?
of a standard FM broadcast station.
Notations:
Notations: ⸹ = frequency deviation (Hz)
BW = Bandwidth fm = modulating frequency (Hz)
⸹max = maximum frequency deviation (Hz) ϕ = phase deviation (rad)
fm(max) = maximum modulating frequency S/N = signal-to-noise ratio
(Hz)
Solution:
Solution: 𝛿 = 𝜙𝑓𝑚
By Carson’s Rule: 𝛿 = (0.3652)(1.5𝑥103 )
BW = 2(⸹max + fm(max)) 𝛿 = 547.8
BW = 2(75 kHz + 15 kHz) = 180 kHz
where: Where:
𝑉𝑛 𝑁
For a standard FM broadcast: sin 𝜙 = =
𝑉𝑠 𝑆
⸹max = 75 kHz
𝑁
fm(max) = 15 kHz 𝜙 = sin−1
𝑆
⸫ The maximum bandwidth of standard 1
FM is 180 kHz. 𝜙 = sin−1
2.8
𝜙 = 20.92°
𝜙 = 0.3652 𝑟𝑎𝑑 Eo = PLL output voltage (V)
⸫ The frequency deviation caused by ⸹ = frequency deviation (Hz)
noise is 547.8 Hz. kf = demodulator sensitivity (Hz/V)

PROBLEM 5: Solution:
In FM broadcasting, what is the carrier 𝛿
𝐸𝑜(𝑝𝑒𝑎𝑘) =
frequency in MHz of a station with a channel 𝑘𝑓
of 285? 65
𝐸𝑜(𝑝𝑒𝑎𝑘) = = 0.325 𝑉
200
Notations: 𝐸𝑜(𝑝𝑒𝑎𝑘)
𝐸𝑜(𝑟𝑚𝑠) =
N = channel number √2
0.325
Fc = carrier frequency (MHz) = 0.23 𝑉
√2
Solution: ⸫ The rms output voltage of the PLL is
𝑁 0.23 V.
𝑓𝑐 = + 47.9
5
285 PROBLEM 8
𝑓𝑐 = + 47.9
5 A phase-locked loop has a VCO with a free-
𝑓𝑐 = 104.0 running frequency of 10 MHz. As the
⸫ The carrier frequency is 104.9 MHz. frequency of the reference input is gradually
raised from zero, the loop locks at 8 MHz and
PROBLEM 6 comes out of lock again at 14 MHz.
In FM broadcasting, what is the channel Determine the lock range.
number of the station with a given carrier of
99.1 MHz? Notations:
fFR = free-running frequency (Hz)
Notations: fLR = lock range (Hz)
N = channel number fCR = capture range (Hz)
fc = carrier frequency fi = input frequency where lock is lost (Hz)
Solution:
N = 5(fc – 47.9) Solution:
N = 5(99.1 – 47.9) 𝑓𝐿𝑅 = 2(𝑓𝐹𝑅 − 𝑓𝑖 )
N = 256 𝑓𝐿𝑅 = 2(14 𝑀𝐻𝑧 − 10 𝑀𝐻𝑧)
⸫ The channel number is 256. 𝑓𝐿𝑅 = 8 𝑀𝐻𝑧
⸫ The lock range is 8 MHz.
PROBLEM 7:
What is the rms output voltage of a PLL FM PROBLEM 9
detector if it uses VCO with proportionality A phase modulator has kp = 2rad/V. What
constant of 200 kHz/V and if it receives an rms voltage of a sine wave would cause a
FM signal with a deviation of 65 kHz sine- peak phase deviation of 30 degrees?
wave modulated?
Notations:
Notations: Em = peak modulating voltage (V)
Φ = peak phase deviation (rad) frequency of the reference input is gradually
kp = sensitivity (rad/V) raised from zero, the loop locks at 12 MHz
and comes out of lock again at 18 MHz.
Solution: Calculate the capture range.
𝜙
𝑘𝑝 =
𝐸𝑚 Notations:
𝜙 fFR = free-running frequency (Hz)
𝐸𝑚 =
𝑘𝑝 fLR = lock range (Hz)
0.52 𝑟𝑎𝑑 fCR = capture range (Hz)
𝐸𝑚 =
𝑟𝑎𝑑 fi = input frequency where lock is lost (Hz)
2
𝑉
𝐸𝑚 = 0.26 𝑉 = 𝐸𝑝𝑒𝑎𝑘
𝐸𝑝𝑒𝑎𝑘 Solution:
𝐸𝑟𝑚𝑠 = From
√2
0.26 𝑓𝐶𝑅
𝐸𝑟𝑚𝑠 = 𝑓𝑖 = 𝑓𝐹𝑅 +
2
√2 𝑓𝐶𝑅 = 2(14 − 12) = 4 𝑀𝐻𝑧
𝐸𝑟𝑚𝑠 = 0.184 𝑉
⸫ The capture range of the PLL is 4
MHz.
Where:
Φ = 30˚ = 0.52 rad
PROBLEM 12
⸫ The rms voltage is 0.184 V.
What is the frequency swing of an FM
broadcast transmitter when modulated
PROBLEM 10
80%?
A system uses a deviation of 100 kHz and a
modulating frequency of 15 kHz. What is the
Notations:
approximate bandwidth?
%M = percent modulation
⸹act = actual frequency deviation (Hz)
Notations:
⸹max = maximum frequency deviation (Hz)
BW = Bandwidth
⸹max = maximum frequency deviation (Hz)
Solution:
fm(max) = maximum modulating frequency 𝛿𝑎𝑐𝑡
(Hz) %𝑀 = 𝑥100%
𝛿𝑚𝑎𝑥
𝛿𝑚𝑎𝑥 (%𝑀)
Solution: 𝛿𝑎𝑐𝑡 =
100%
By Carson’s Rule: (75 𝑘𝐻𝑧)(80%)
BW = 2(⸹max + fm(max)) 𝛿𝑎𝑐𝑡 =
100%
BW = 2(100 kHz + 15 kHz) = 230 kHz 𝛿𝑎𝑐𝑡 = 60 𝑘𝐻𝑧
⸫ The approximate bandwidth is 230
kHz. where for FM broadcast
⸹max = 75 kHz
PROBLEM 11 ⸫ The frequency swing of an FM
A phase-locked loop has a VCO with a free- broadcast transmitter is 60 kHz.
running frequency of 14 MHz. As the
PROBLEM 13 Notations:
Determine the modulating index of a %M = percent modulation
standard FM broadcast having a hypothetical ⸹act = actual frequency deviation (Hz)
maximum carrier frequency deviation of ±12 ⸹max = maximum frequency deviation (Hz)
kHz and a maximum modulating frequency
of 4 kHz. Solution:
Notations: 𝛿𝑎𝑐𝑡
%𝑀 = 𝑥100%
m = modulating index 𝛿𝑚𝑎𝑥
⸹max = maximum frequency deviation (Hz) 𝐶. 𝑆.
𝛿𝑎𝑐𝑡 =
fm = modulating frequency (Hz) 2
%𝑀 𝛿𝑎𝑐𝑡 𝐶. 𝑆.
𝑀= = =
100 𝛿𝑚𝑎𝑥 2𝛿𝑚𝑎𝑥
Solution:
𝛿𝑚𝑎𝑥 𝐶. 𝑆. = 2𝑀𝛿𝑚𝑎𝑥
𝑚= 𝐶. 𝑆. = 2(0.80)(75) = 120 𝑘𝐻𝑧
𝑓𝑚
12 𝑘𝐻𝑧 ⸫ The carrier swing is 120 kHz.
𝑚= =3
4 𝑘𝐻𝑧
⸫ The modulation index of a standard PROBLEM 16
FM broadcast is 3. An FM signal has a center frequency of 100
MHz but is swinging between 100.01 MHz
PROBLEM 14 and 99.999 MHz at a rate of 100 times per
What is the modulation index of an FM second. What is the modulation index of the
transmitter whose frequency deviation is 50 signal?
kHz, while audio frequency is 10 kHz?
Notations:
Notations: m = modulation index
m = modulating index 𝛿 = frequency deviation (Hz)
⸹act = actual frequency deviation (Hz) fm = modulating frequency (Hz)
fm = modulating frequency (Hz)
Solution:
Solution:
𝛿𝑎𝑐𝑡 ⸫ The intelligence frequency is equal to
𝑚=
𝑓𝑚 the rate of change of the carrier
50 𝑘𝐻𝑧 frequency.
𝑚= =5
10 𝑘𝐻𝑧
⸫ The modulation index of an FM Therefore, fm = 100 Hz.
transmitter is 5.
𝐶. 𝑆.
𝛿
PROBLEM 15 𝑚= = 2
𝑓𝑚 𝑓𝑚
The carrier swing necessary to provide 80%
100.01 − 99.99
modulation in the FM broadcasting band is (
2
) 𝑀𝐻𝑧
𝑚= = 100
________. 100 𝐻𝑧

⸫ The modulation index is 100.


PROBLEM 17 @ f = 30 Hz
If the frequency fed to the pre-amplifier of a m = 2500
basic transmitter with multipliers is 𝐵𝑊 = 2(𝑓𝑚 + 𝛿) ≈ 2(𝛿) = 150 𝑘𝐻𝑧
composed of a pair of triplers and doubler @f = 15 kHz
multipliers is 198 MHz. What frequency m=5
should the oscillator operate? 𝐵𝑊 = 2(𝑓𝑚 + 𝛿) = 2(15 + 75) = 180 𝑘𝐻𝑧

Notations: ⸫ The maximum bandwidth is 180 kHz.


fout = output of multiplier circuits (Hz)
fo = output of oscillator circuits (Hz) PROBLEM 19
A portable radio transmitter has to operate
Solution: at temperatures from -5 degrees to 35
degrees C. If its signal is derived from a
crystal oscillator with a temperature
coefficient of +1 ppm per degree
centigrade, and it transmit at exactly 145
MHz at 20 degrees C, find the transmitting
Multiplier Circuits frequency at 35 degrees C.

𝑓𝑜𝑢𝑡 = (𝑓𝑜 )(3)(3)(2) = 198 𝑀𝐻𝑧 Notations:


𝑓𝑜𝑢𝑡 198 𝑀𝐻𝑧 fT = operating frequency at temperature T
𝑓𝑜 = = = 11 𝑀𝐻𝑧
18 18 fo = operating frequency at reference
temperature T
⸫ The operating frequency of the k = temperature coefficient per degree
oscillator is 11 MHz. centigrade
Solution:
PROBLEM 18
For commercial FM broadcasting the 𝑓𝑇 = 𝑓𝑜 + 𝑘𝑓𝑜 (𝑇 − 𝑇𝑜 )
maximum permissible range in modulation 𝑓𝑇 = 145 𝑀𝐻𝑧 + (1 × 10−6 )(145 × 106 )(35
index is 5 – 2500. What is the maximum − 20) = 145.002175 𝑀𝐻𝑧
bandwidth needed for this system?
Note: The term parts per million (ppm) tells
Notations: how many cycles (Hz) a crystal may differ
BW = Bandwidth (Hz) from its designated frequency for every
𝛿 = frequency deviation (Hz) 1,000,000 Hz of frequency. A 100-ppm
fm = modulating frequency (Hz) deviation on a 10 MHz crystal means that the
actual frequency could be 10 x 10 ppm =
Solution: 1000 Hz plus or minus 10 MHz.
⸫The transmitting frequency is
For commercial FM broadcasting 145.002175 MHz.
𝛿𝑚𝑎𝑥 = 75 𝑘𝐻𝑧
𝛿
𝑚=
𝑓𝑚
PROBLEM 20 Notations:
What FM channel is 107.5 MHz carrier? δ = frequency deviation (Hz)
kf = sensitivity of the modulator (kHz/V)
Notations: Vm = peak amplitude of the modulating
N = channel number signal (V)
fC = carrier frequency (MHz)
Solution:
Solution:
𝑘𝐻𝑧
𝛿 = 𝑘𝑓 𝑉𝑚 = 20 (3.54 𝑉) = 70.8 𝑘𝐻𝑧
𝑁 = 5(𝑓𝑐 − 47.9) 𝑉
𝑁 = 5(107.5 − 47.9) = 298
Where:
⸫ The FM channel is 298. 𝑉𝑚 = √2(2.5) = 3.54 𝑉

PROBLEM 21 ⸫ The frequency deviation of the FM


The instantaneous value of the modulating modulator is 70.8 kHz.
signal to an FM modulator is -2V, sensitivity
kf = 25 kHz/V and operating at a carrier PROBLEM 23
frequency of 88 MHz. Determine the output Find the deviation ratio of an FM broadcast
frequency of the modulator. transmitter if the frequency of the
modulation signal varies from 30 Hz to 20
Notations: kHz.
fsig =output frequency of the modulator (Hz)
fo = carrier frequency (Hz) Notations:
kf = sensitivity (Hz/V) DR = Deviation Ratio
Em = modulating signal voltage (V) δmax= maximum frequency deviation (Hz)
fmax = maximum modulating frequency (Hz)
Solution:
Solution:
𝑓𝑠𝑖𝑔 = 𝑓𝑜 + 𝑘𝑓 𝐸𝑚
103 𝐻𝑧 𝛿𝑚𝑎𝑥 75 𝑘𝐻𝑧
𝑓𝑠𝑖𝑔 = (88 × 106 𝐻𝑧) + (25 × ) (−2𝑉) 𝐷𝑅 = = = 3.75
𝑉 𝑓𝑚𝑎𝑥 20 𝑘𝐻𝑧
𝑓𝑠𝑖𝑔 = 87.95 𝑀𝐻𝑧
⸫ The deviation ratio of an FM
⸫ The output frequency of the broadcast transmitter is 3.75.
modulator is 87.95 MHz.
PROBLEM 24
The output signal of a PM demodulator is
PROBLEM 22
Calculate the frequency deviation of an FM 0.54 V. What is the sensitivity of the
demodulator if the phase deviation is 30°?
modulator that has kf =20 kHz/V and
operates at a carrier frequency of 99.1 MHz
modulated by a 2.5 Vrms sine wave.
Notations: Then,
ko = sensitivity of the phase modulator 𝛿
𝜙𝑚𝑎𝑥 =
(rad/V) 𝑓𝑚
𝜋
Φ = phase deviation (rad) 𝛿 = 𝑓𝑚 𝜙𝑚𝑎𝑥 = (300 𝐻𝑧) ( ) = 314 𝐻𝑧
Vm = peak amplitude of the modulating 3
signal (V)
⸫ The maximum frequency deviation of
Solution:
the system is 314 Hz.

Convert phase deviation from degrees to


PROBLEM 26
radian.
An FM communication system has a
sensitivity of kf = 3 kHz/V. How much phase
360° = 2𝜋 𝑟𝑎𝑑
2𝜋 𝑟𝑎𝑑 × 30 𝜋 deviation does it produce with a sine wave
30° = = 𝑟𝑎𝑑 input of 3 V peak at a frequency of 2 kHz?
360 6
𝜙
𝑘𝑝 = Notations:
𝑉𝑚
𝜋
( ) 𝑟𝑎𝑑 mf = FM modulation index
𝑟𝑎𝑑
𝑘𝑝 = 6 = 0.97 δ = frequency deviation (Hz)
0.54 𝑉
fm = modulating frequency (Hz)
⸫ The sensitivity of the modulator is Φ = phase deviation (rad)
0.97 rad/V.
Solution:
PROBLEM 25
𝛿 = 𝑘𝑓 𝑉𝑚
A PM transmitter produces a maximum
𝑘𝐻𝑧
phase shift of π/3 rad for a modulating 𝛿 = (3 ) (3𝑉) = 9 𝑘𝐻𝑧
𝑉
frequency of 300 Hz. What is the maximum
For a PM system, modulation index is equal
frequency deviation of the system?
to peak phase deviation.
𝛿
Notations: 𝑚𝑓 = = 𝜙𝑝 = 𝑚𝑝
𝑓𝑚
mf = FM modulation index 9 𝑘𝐻𝑧
δ = frequency deviation (Hz) 𝜙=
2 𝑘𝐻𝑧
= 4.5 𝑟𝑎𝑑
fm = modulating frequency (Hz)
Φ = phase deviation ⸫ The peak phase deviation is 4.5 rad.

Solution: PROBLEM 27
𝛿
From: 𝑚𝑓 = 𝑓𝑚
An FM broadcast transmitter rated output
For a PM system, modulation index is equal power is 5 W. What is the peak voltage
to peak phase deviation. developed across a 50 ohms resistive load?

𝑚 = 𝜙𝑚𝑎𝑥 Notations:
PT = output power (W)
VT = peak voltage across the load (V)
RL = load resistance (ohm) Notations:
PT = total power transmitted (W)
Solution: PC = carrier power (W)
J = Bessel coefficient of the sidebands
𝑉𝑇 2
𝑃𝑇 =
𝑅𝐿 Solution:
𝑉𝑇 = √𝑃𝑇 𝑅𝐿 = √(5)(50) = 15.8 𝑉𝑟𝑚𝑠
From
𝑉𝑇 (𝑝𝑒𝑎𝑘) = 𝑉𝑇 √2 = 22.34 𝑉
𝑃𝑇 = 𝑃𝐶 [𝐽𝑜 2 + 2(𝐽1 2 + 𝐽2 2 + 𝐽3 2 + ⋯ )]
⸫ The peak voltage developed across Therefore:
the load is 22.34 V. 𝑃2 = 2𝐽2 2 𝑃𝐶 = 2(0.49)2 (7) = 3.4 𝑊

PROBLEM 28 Note: PT = PC if all the J coefficients are


An FM system has a modulation index of 3 considered.
and its corresponding Bessel coefficient; Jo
= -0.26; J1 = 0.34; J2 = 0.49; J3 = 0.31. ⸫ The power of the second sideband is
Find the RMS carrier voltage if the total 3.4 W.
power is 5 W, developed across a 50 ohms
resistive load. PROBLEM 30
An FM broadcast transmitter has a deviation
Notations: of 25 kHz and a modulating frequency of 8
VT = voltage level at the load (V) kHz. Calculate the power that is
VC = voltage level of the carrier (V) unaccounted, if the total power PT is 10 W.
J = Bessel coefficient of the sidebands Jo = -0.26; J1 = 0.34; J2 = 0.49; J3 = 0.31

Solution: Notations:
PTr = power transmitted (W)
mf =3 P0, P1, P2 ... = carrier power, first sideband
Jo = -0.26; J1 = 0.34; J2 = 0.49; J3 = 0.31 power, second sideband power ... (W)
𝑉𝐶 = 𝐽𝑜 𝑉𝑇 = |𝐽𝑜 |𝑉𝑇 = (0.26)(15.8) = 4.11 𝑉 J = Bessel coefficient of the sidebands
Where: PT = total power transmitted (W)
𝑉𝑇 2 Px = unaccounted power (W)
𝑃𝑇 =
𝑅𝐿
Solution:
𝑉𝑇 = √𝑃𝑇 𝑅𝐿 = √(5)(50) = 15.8 𝑉𝑟𝑚𝑠
⸫ The RMS carrier voltage is 4.11 V.
m=3
Jo = -0.26; J1 = 0.34; J2 = 0.49; J3 = 0.31
PROBLEM 29
J3 = 0.31
Suppose the total power an FM signal is 7
𝑃𝑇𝑟 = 𝑃0 + 2(𝑃1 + 𝑃2 + 𝑃3 )
W. What is the power of the second
𝑃𝑇𝑟 = 𝐽0 2 𝑃𝑇 + 2(𝐽1 2 𝑃𝑇 + 𝐽2 2 𝑃𝑇 + 𝐽3 2 𝑃𝑇
sideband if the modulation index is 3? Jo = -
𝑃𝑇𝑟 = (0.26)2 (10) + 2[(0.34)2 (10)
0.26; J1 = 0.34; J2 = 0.49; J3 = 0.31
+ (0.49)2 (10) + (0.31)2 (10)]
𝑃𝑇𝑟 = 9.712 𝑊 𝑉𝑠 𝛿𝑠 75 𝑘𝐻𝑧
= = = 30
Thus: 𝑉𝑛 𝛿𝑛 2.5 𝑘𝐻𝑧
𝑃𝑥 = 𝑃𝑇 − 𝑃𝑇𝑟 = 10 − 9.712 = 0.288 𝑊 𝑆
( )𝑜𝑑𝐵 = 20𝑙𝑜𝑔30 = 29.5 𝑑𝐵
𝑁
⸫ The unaccounted power is 0.288 W.
∴ The approximate signal-to-noise
PROBLEM 31 ratio at the detector output is 34 dB.

A certain FM station transmitted a PROBLEM 32


modulating frequency of 25 kHz and a An FM broadcast station transmits a 3 kHz
frequency deviation of 75 kHz. What is the test tone, and a frequency deviation of 15
approximate signal-to-noise ratio at the kHz. Determine the modulation index.
detector output if the signal-to-noise ratio at
the input to the receiver detector is 20 dB? Notations:

Notations: mf = FM modulation index


Vs = signal voltage (V) δ= frequency deviation (Hz)
Vn = noise voltage (V) fm = modulating frequency (Hz)
Φ = peak phase deviation (rad)
δ= peak frequency deviation (Hz) Solution:
mf= FM modulation
fm= modulating frequency (Hz) 𝛿 15 𝑘𝐻𝑧
𝑚𝑓 = = =5
𝑓𝑚 3 𝑘𝐻𝑧
Solution: ∴ The modulation index of an FM
𝑆 broadcast station is 5.
𝑉𝑠 ( ) 20
−1 𝑁 𝑑𝐵
( ) = 𝑙𝑜𝑔 = 𝑙𝑜𝑔−1 = 10 PROBLEM 33
𝑉𝑛 20 20
𝑉𝑛 An FM signal with modulation index of 4 and
= 0.1 modulating frequency of 8 kHz. What is the
𝑉𝑠
frequency deviation of the signal?
Since Vs > Vn
Notations:
𝑉𝑛
𝛷= = 0.1 𝑟𝑎𝑑
𝑉𝑠 mf = FM modulation index
δ = frequency deviation (Hz)
Thus:
fm = modulating frequency (Hz)
mfm = 0.1
Solution:
δn = mfm (fm) 𝛿
𝑚𝑓 = ⇨ 𝛿 = 𝑚𝑓 𝑓𝑚 = 4(8𝑘𝐻𝑧) = 32 𝑘𝐻𝑧
But noise can interfere only if the frequency 𝑓𝑚
of noise is the same as the frequency of the ∴ The frequency deviation is 32 kHz.
signal thus,

𝛿𝑛 = 𝑚𝑓𝑛 (𝑓𝑚 )= (0.1) (25kHz) = 2.5 kHz


PROBLEM 34 PROBLEM 36
The maximum instantaneous frequency of
A certain FM modulator has Kf = 25KHz/V.
the modulated signal is 95.05 MHz. If a 5 KHz
Suppose the peak amplitude of the
signal modulates a 95 MHz carrier, calculate
modulating signal is 3 V at a frequency of 3
the modulation index of the signal.
KHz, determine the modulation index of the
signal.
Notations:
Notations: fins = instantaneous frequency (Hz)
mf= FM modulation index fc= carrier frequency (Hz)
δ= frequency deviation (Hz) δ= frequency deviation (Hz)
fm= modulating frequency (Hz) m = modulating index
Vm = peak amplitude of the modulating fm= modulating frequency (Hz)
signal (V)
Solution:
Solution: 𝑓𝑖𝑛𝑠 = 𝑓𝑐 + 𝛿 ⇨ 𝛿 = 𝑓𝑖𝑛𝑠 − 𝑓𝑐
𝑘𝐻𝑧 𝛿 = 95.05 𝑀𝐻𝑧 − 95 𝑀𝐻𝑧 = 50𝑘𝐻𝑧
𝛿 = 𝑘𝑓 𝑉𝑚 = (25 ) (3) = 75 𝑘𝐻𝑧 𝛿 50 𝑘𝐻𝑧
𝑉
𝛿 75 𝑘𝐻𝑧 𝑚= = = 10
𝑓𝑚 5 𝑘𝐻𝑧
𝑚𝑓 = = = 25
𝑓𝑚 3 𝑘𝐻𝑧 ∴ The modulation index of the signal is
∴ The modulation index of the signal is 10.
25.
PROBLEM 37
PROBLEM 35 A PM signal has a RMS signal of 3V at a
The output frequency of the modulator frequency of 3 kHz. If the modulation index
increases by 500 KHz. What is the dc voltage of the signal is 15, determine the sensitivity
applied to the FM modulator, if the deviation of the modulator.
sensitivity is 25 KHz/V?
Notations:
Notations: mp = PM modulation index
δ= frequency deviation (Hz) Φp= peak phase deviation (rad)
k = sensitivity Kp= sensitivity (rad/V)
Vm= peak amplitude of the modulating Vm = peak amplitude of the modulating
signal (V) signal (V)
Solution:
Solution: 𝑚𝑝 = 𝛷𝑝 = 𝑘𝑝 𝑉𝑚
𝛿 𝛿 50 𝑘𝐻𝑧 𝑚𝑝 15 𝑟𝑎𝑑
𝑘=± ⇨𝑉=+ = =2𝑉 𝑘𝑝 = = = 3.54
𝑉𝑚 𝑘 𝑘𝐻𝑧 𝑉𝑚 3(√2) 𝑉
25 𝑉
∴ The dc voltage applied to the FM ∴ The sensitivity of the modulator is
modulator is 2 V. 3.54 rad/V.
PROBLEM 38 Solution:
The modulation index of the PM signal is 2. δ = k p Vm fm
If a modulating signal that has amplitude of δ 75 𝑘𝐻𝑧
𝑉𝑚 = = = 100𝑚𝑉
50 mV at a frequency of 3 kHz phase 𝑘𝑝 𝑓𝑚 𝑟𝑎𝑑
(50 𝑉 ) (15 𝑘𝐻𝑧)
modulates a 90 kHz carrier signal, what
Where:
would be the new modulation index of the
𝛿 = 𝛷𝑝 𝑓𝑚 ⇨ 𝛷𝑝 = 𝑘𝑝 𝑉𝑚
signal? Assume that the amplitude of the
∴ The peak amplitude of the modulating
modulating signal is increased by 50%.
signal is 100 mV.
Notations:
mp= PM modulation index PROBLEM 40
Kp= sensitivity (rad/V) With a maximum frequency deviation of 75
Φ= peak phase deviation (rad) kHz, what is the maximum phase deviation
that can be present in an FM radio broadcast
Vm= peak amplitude of the modulating
signal (V) signal, assuming it transmits a baseband
frequency of 50 Hz to 20 kHz?
Solution:
Notations:
𝛷𝑝 2 𝑟𝑎𝑑 mf(max)= maximum modulation index
𝑉𝑚 = = = 40 δmax= maximum frequency deviation (Hz)
𝑉𝑚(𝑜𝑙𝑑) 0.05 𝑉 𝑉
𝑟𝑎𝑑 fm= modulating frequency
𝑚𝑝 = 𝑘𝑉𝑚(𝑛𝑒𝑤) = (40 ) (0.05)(1.5) = 3
𝑉
Where: Solution:
𝑉𝑚(𝑛𝑒𝑤) = 𝑉𝑚(𝑜𝑙𝑑) + 0.05𝑉𝑚(𝑜𝑙𝑑)
∴ The new modulation index is 3. The maximum modulation index of the
signal occurs when frequency of the
PROBLEM 39 modulating signal is minimum, thus
A sine wave of frequency 15 kHz frequency- 𝛿𝑚𝑎𝑥 75 𝑘𝐻𝑧
𝑚𝑓(max) = = = 1500
modulates a carrier at 101.1 MHz. The peak 𝑓𝑚𝑖𝑛 50 𝐻𝑧
frequency deviation is 75 kHz. Determine the ∴ The maximum phase deviation phase
peak amplitude of the modulating signal if modulation index is 1500 rad.
the sensitivity of the modulator is 50 rad/V.
PROBLEM 41
Notations: The phase shift of a phase modulated signal
mp= PM modulation index is 86°. What is the modulation index of the
Φp= peak phase deviation (rad) said signal?
Kp= sensitivity (rad/V)
Vm= peak amplitude of the modulating Notations:
signal (V) ΔΦ= phase shift (rad)
δ= frequency deviation (Hz) Mp = PM modulation index
Solution: 𝛿 100 𝑘𝐻𝑍
𝜋 𝑓𝑚 = = = 20 𝑘𝐻𝑧
𝛥𝛷 = 86° 𝑥 𝑚 5
180𝑜 𝐵𝑊 = 2(𝛿 + 𝑓𝑚 ) = 2(100𝑘𝐻𝑧 + 20 𝑘𝐻𝑧)
𝛥𝛷 = 1.5 𝑟𝑎𝑑 = 240 𝑘𝐻𝑧
For PM system, modulation index is equal to ∴ The bandwidth using Carson’s rule is
the phase deviation of the signal. 240 kHz.
mp = Φp = 1.5
∴ The modulation index is 1.5. PROBLEM 44
An FM station transmits 100 W of power. If
PROBLEM 42 the modulation index of the signal is 7, how
Determine the modulation index of an FM much power is transmitted at the carrier
signal, given that the modulating frequency frequency if Jo= 0.4?
is 25 kHz and frequency deviation of 75
kHz. Notations:
Po = carrier power (W)
Notations: Jo = carrier Bessel coefficient
mf = FM modulation index PT = total transmitted power (W)
δ = frequency deviation (Hz)
fm = modulating frequency (Hz) Solution:
From the Bessel Function Table:
Solution: m = 7; Jo = 0.4
𝑃𝑜 = 𝐽𝑜2 𝑃𝑇
𝛿 𝑃0 = (0.4)2 (100𝑊) = 16 𝑊
𝑚𝑓 =
𝑓𝑚 ∴ The power transmitted at the carrier
75 𝑘𝐻𝑧 frequency is 16 W.
𝑚𝑓 = =3
25 𝑘𝐻𝑧
∴ The modulation index of the signal is
PROBLEM 45
3.
A 50 ohms dummy load is used to test a
certain FM station. Suppose the measured
PROBLEM 43
power is 12.5 W, what is the voltage level of
Suppose a certain FM signal uses a frequency
the second sideband if a frequency of 5 kHz
deviation of 100 kHz and a modulating index
test tone is used and a 10 kHz frequency
of 5. Calculate the bandwidth using Carson’s
deviation? Note from Bessel Table m=2; J2=
rule.
0.35
Notations:
BW= bandwidth (Hz)
Notations:
δ= maximum frequency deviation (Hz)
VT = voltage measured at the load (V)
fm= maximum modulating frequency (Hz)
PT = total transmitted power (W)
Mf= FM modulation index
R = load resistance (Ω)

Solution:
Solution:
𝛿
𝑚𝑓 = VT = √PT R
𝑓𝑚
VT = √(12.5)(50) = 25𝑉 Notations:
V2 = J2 VT Pc, P1, P2... = carrier power, first sideband
V2 = 0.35 (25V) = 8.75 VRMS power, second sideband power …
∴ The voltage level of the second
sideband is 𝟖. 𝟕𝟓 𝐕𝐑𝐌𝐒 . J = Bessel coefficient of the sidebands
PTr = transmitted power (W)
PROBLEM 46
Solution:
An FM receiver can sufficiently receive only
From 𝑃𝑇 = 𝑃𝑐 [𝐽02 + 2(𝐽12 + 𝐽22 + 𝐽32 + ⋯ )]
the carrier and the first two sets of
sidebands. If the total power of the signal is 𝑃0 = 𝐽02 𝑃𝑇
20W (ignore any losses in the signal), 𝑃0 = (0.22)2 (10) = 484 mW
deviation of 25 kHz, and a modulation index
∴ The power transmitted at the carrier
of 10, what is the percentage of the total
frequency is 484 mW.
signal power will it receive?
PROBLEM 48
Notations:
The calculated bandwidth using Carson’s rule
Pc, P1, P2... = carrier power, first sideband
us 40 kHz if the modulating frequency of the
power, second sideband power …
signal is 5 kHz. Using the same signal, what
J = Bessel coefficient of the sidebands is the bandwidth if the sidebands with more
PTr = transmitted power (W) than 1% of the signal voltage are
considered?
Solution:
Notations:
𝑃𝑇 = 𝑃𝑐 [𝐽02 + 2(𝐽12 + 𝐽22 + 𝐽32 + ⋯ )]
N = number of significant sidebands
Thus;
Solution:
𝑃𝑇𝑟 = 𝑃𝑐 (𝐽02 + 2𝐽12 + 2𝐽22 )
BW = 2(𝛿 + 𝑓𝑚 )
𝑃𝑇𝑟 = (0.22)2 + 2(0.58)2 + 2(0.35)2
BW
𝑃𝑇𝑟 = 0.9622 × 100% 𝛿= − 𝑓𝑚
2
𝑃𝑇𝑟 = 96.62% of 𝑃𝑐 40 kHz
𝛿= − 5 kHz = 15 kHz
∴ 96.62% of the carrier will be 2
transmitted. 𝛿
𝑚=
𝑓𝑚
PROBLEM 47
15 kHz
An FM receiver sufficiently receives only the 𝑚= =3
5 kHz
carrier and the first two sets of sidebands. If
the total power of the signal is 10W (ignore Using Bessel Function Table
any losses in the signal), deviation is 25 kHz, BW = 2𝑓𝑚 𝑁
and the modulation index is 10, how much
BW = 2(5 kHz)(6) = 60 kHz
power is transmitted at the carrier
frequency? where: 𝑚 = 3; 𝑁 = 6
∴ The bandwidth of the signal is 60 kHz. Notations:
PROBLEM 49 ϕN = phase deviation due to noise (rad)
An FM transmitter operates with a carrier VN = noise voltage (V)
power of 10 W. What is the total carrier
VS = signal voltage (V)
power if the modulation index of the signal is
5? S/N = signal-to-noise power ratio

Notations: Solution:
𝑉𝑁
Vc = carrier voltage (V) sin 𝜙𝑁 =
𝑉𝑆
P0 = carrier power (W)
𝑉𝑁
For ≪1
R = load resistance (Ω) 𝑉𝑆

𝑉𝑁
δ = maximum frequency deviation (Hz) 𝜙𝑁 ≈
𝑉𝑆
fm = maximum modulating frequency (Hz)
𝑆 𝑉𝑆
mf = FM modulation index ( ) = 20 log
𝑁 𝑖 𝑉𝑁
Solution: 𝑆 1
( ) = 20 log
𝑉𝑐2 𝑁 𝑖 𝜙𝑁
𝑃0 =
𝑅 𝑆 1
( ) = 20 log = 40 dB
𝑁 𝑖 0.01
𝑉𝑐 = √𝑃0 𝑅
∴ The signal-to-noise ratio is 40 dB.
𝑉𝑐 = √10(1) = 3.16 V
PROBLEM 51
Using Bessel Function Table
An FM receiver detects a 100 mV signal. A
at m = 5; J0 = 0.18 spurious signal is also detected by the
𝑉𝑐 receiver that resulted to an S/N of 40 dB at
𝑉𝑇 =
𝐽0 the detector’s output. Determine the
3.16 minimum S/N ratio at the detector’s output if
𝑉𝑇 = = 17.57 Vrms the modulating frequency is 2 kHz and the
0.18
frequency deviation of 50 kHz.
𝑉𝑇2 (17.57 V)2
𝑃𝑇 = = = 308 W
𝑅 1Ω Notations:
∴ THE TOTAL SIGNAL POWER IS 308 W. ϕN = phase deviation due to noise (rad)
PROBLEM 50 ϕs = phase deviation due to signal (rad)
The maximum phase deviation given to the VN = noise voltage (V)
signal by the noise voltage is 0.01 radian.
VS = signal voltage (V)
What is the signal-to-noise ratio at the
detector’s input? S/N = signal-to-noise power ratio
fm = modulating frequency (Hz)
Solution: Solution:
𝑆 𝛿𝑆 𝑓𝑠𝑖 = 𝑓𝑠 + 2𝑓𝑖
( ) = 20 log
𝑁 0 𝛿𝑁
𝑓𝑠𝑖 = 99.1 + 2(10.7) = 120.05 MHz
𝛿𝑆 𝑆
= antilog IR = √1 + 𝑄 2 𝜌2
𝛿𝑁 𝑁
𝛿𝑆 40 𝑓𝑠𝑖 𝑓𝑠
= antilog = 100 𝜌= −
𝛿𝑁 20 𝑓𝑠 𝑓𝑠𝑖

𝛿𝑆 50 kHz 120.5 99.1


𝛿𝑁 = = = 500 Hz 𝜌= − = 0.393
100 100 99.1 120.5

𝛿𝑁 = 𝜙𝑁 𝑓𝑚 IR = √1 + 452 ⋅ 0.3932
𝛿𝑁 IRdB = 20 log 17.74 = 25 dB
𝜙𝑁 =
𝑓𝑚 ∴ The image frequency rejection ratio in
500 Hz dB is 25 dB.
𝜙𝑁 = = 0.25 rad
2 kHz
PROBLEM 53
𝑉𝑁
sin 𝜙𝑁 =
𝑉𝑆
≈ 0.25 An FM detector receives an FM signal with a
deviation of 15 kHz. What is the peak-to-
Thus, peak output voltage of the detector if its
𝑆 1 sensitivity is 75 μV/Hz?
( ) = 20 log
𝑁 𝑖 𝜙𝑁 Notations:
𝑆 1
( ) = 20 log = 12 dB Vm = peak amplitude of the modulating
𝑁 𝑖 0.25
signal (V)
∴ The signal-to-noise ratio at the
kf = sensitivity (V/Hz)
detector’s output is 12 dB.
δ = frequency deviation (Hz)
PROBLEM 52
Vp-p = peak-to-peak output voltage (V)
An FM receiver is tuned at 99.1 MHz. What is
the image frequency rejection ratio if the Solution:
receiver consists of a single tuned circuits 𝑉𝑚 = 𝑘𝑓 𝛿
with Q of 45?
μV
Notations: 𝑉𝑚 = (75 ) (15 kHz)
Hz
IR = image frequency rejection ratio 𝑉𝑚 = 1.125 𝑉
fs = signal frequency (Hz) 𝑉𝑝−𝑝 = 2𝑉𝑝
fsi = image frequency (Hz)
𝑉𝑝−𝑝 = 2(1.125 V) = 2.25 V
fi = intermediate frequency (Hz)
∴ The peak-to-peak output voltage of
Q = quality factor the detector is 2.25 V.
PROBLEM 54 𝑘𝑜𝑝𝑡 = 1.5𝑘𝑐
An FM receiver uses PLL to detect the signal. 𝑘𝑜𝑝𝑡 = 1.5(0.0239) = 0.03585
What is the sensitivity of the VCO in the PLL
BW = 𝑘𝑓0
if the output of the detector is 1.25 Vp from
an FM signal that is modulated to 10 kHz BW = 0.03585(10.7 MHz) = 383.6 kHz
deviation by sine wave? ∴ The bandwidth of the tuner is 383.6
Notations: kHz.

Vm = peak amplitude of the modulating PROBLEM 56


signal (V) An FM superheterodyne receiver is tuned to
kf = sensitivity (V/Hz) a frequency of 88 MHz. What is the local
oscillator frequency if low side injection is
δ = frequency deviation (Hz)
used at the mixer?
Solution:
Notations:
𝛿
𝑘𝑓 = fLO = local oscillator frequency (Hz)
𝑉𝑚
fRF = signal frequency (Hz)
10 kHz
𝑘𝑓 = = 8 kHz/V
1.25 V fIF = intermediate frequency (Hz)
∴ The sensitivity of the receiver is 8 Solution:
kHz/V
𝑓𝐿𝑂 = 𝑓𝑅𝐹 ± 𝑓𝐼𝐹
PROBLEM 55
For low side injection
An FM receiver uses a tuned transformer
𝑓𝐿𝑂 = 𝑓𝑅𝐹 − 𝑓𝐼𝐹
operating at 10.7 MHz. What is the
bandwidth using the optimum coupling factor 𝑓𝐿𝑂 = 88 MHz − 10.7 MHz
if the primary circuit has a Q of 50 and the 𝑓𝐿𝑂 = 77.3 MHz
secondary has Q of 35?
∴ The local oscillator frequency is 77.3
Notations: MHz.
kc = critical coupling coefficient PROBLEM 57
kopt = optimum coupling coefficient A certain FM receiver has a blocking dynamic
Q = quality factor range of 75 dB. What is the maximum signal
that this receiver can handle, if the desired
BW = bandwidth (Hz)
signal has a level of 25 μV?
Solutions:
Notation:
1
𝑘𝑐 = DR = Dynamic Range
√𝑄𝑝 𝑄𝑠
Vmax = strongest signal (V)
1
𝑘𝑐 = = 0.0239 Vmin = weakest signal (V)
√(50)(35)
Solution: fs = signal frequency (Hz)
Vmax fsi = image frequency (Hz)
DRdB = 20 log ( )
Vmin
fi = intermediate frequency (Hz)
DRdB Q = quality factor
Vmax = Vmin antilog ( )
20
Solution:
75
Vmax = (25 μV) (antilog ( ))
20 IR = √1 + 𝑄 2 𝜌2
Vmax = 140.6 mV IR = √1 + (60)2 (0.41)2 = 24.62
∴ The maximum signal is 140.6 mV. Where:
PROBLEM 58 𝑓𝑠𝑖 𝑓𝑠
𝜌= −
An FM broadcast receiver with high side 𝑓𝑠 𝑓𝑠𝑖
injection is tuned to a station at 93.5 MHz. 117.1 95.7
𝜌= − = 0.41
What is the image frequency? 95.7 117.1
Notations: 𝑓𝑠𝑖 = 𝑓𝑠 + 2𝑓𝑖

fLO = local oscillator frequency (Hz) 𝑓𝑠𝑖 = 95.7 + 2(10.7) = 117.1 MHz

fRF = signal frequency (Hz) For two uncouples tuned circuit

fIM = image frequency (Hz) IR 𝑇 = IR2


Solution: IR 𝑇 dB = 20 log(24.62)2 = 55.7 dB

From ∴ The image frequency rejection ratio is


𝑓𝐼𝑀 = 𝑓𝑅𝐹 ± 2𝑓𝐼𝐹 55.7 dB.

For high-side injection PROBLEM 60

𝑓𝐼𝑀 = 𝑓𝑅𝐹 + 2𝑓𝐼𝐹 An FM broadcast receiver turns from 88 to


108 MHz. The IF is 10.7 MHz. The total
𝑓𝐼𝑀 = 93.5 MHz + 2(10.7 MHz) = 114.9 MHz oscillator uses a variable capacitor with a
∴ The image frequency is 114.9 MHz. minimum value of 125 pF. Suppose the mixer
uses high side injection, what is the value of
PROBLEM 59
the inductor so that the local oscillator will
A superheterodyne receiver uses two tune the receiver to the highest frequency on
uncoupled tuned circuits, each with a Q of the band when the capacitor is minimum?
60. What is the image frequency rejection
Notations:
ratio of the receiver if the signal frequency is
95.7 MHz and the IF is 10.7 MHz? Assume fLO = local oscillator frequency (Hz)
high side injection is used in the mixer. fRF = signal frequency (Hz)
Notations: fIF = intermediate frequency (Hz)
IR = image frequency rejection ratio
Solution: transmitting frequency when the
temperature is 35°C.
𝑓𝐿𝑂 = 𝑓𝑅𝐹 + 𝑓𝐼𝑅
𝑓𝐿𝑂 = 108 MHz + 10.7 MHz = 118.7 MHz Notations:

1 fT = operating frequency at temperature T


𝑓𝐿𝑂 =
2𝜋√𝐿𝐶 fo = operating frequency at reference
1 temperature To
𝐿=
(2𝜋𝑓)2 𝐶
k = temperature coefficient per degree
1
𝐿= Solution:
(2𝜋 ⋅ 118.7 × 106 )2 (125 × 10−12 )
= 14.4 nH At a temperature of 35°C:
∴ The inductance of the tuner is 14.4 𝑓𝑇 = 𝑓𝑜 + 𝑘𝑓𝑜 (𝑇 − 𝑇𝑜 )
nH.
𝑓𝑇 = 101 𝑀𝐻𝑧 + 101 𝑀𝐻𝑧(−1𝑥10−6 )(35 −
PROBLEM 61 20)
A PLL circuit uses a crystal oscillator 𝑓𝑇 = 100.998485 𝑀𝐻𝑧
accurate within 0.001%. What is the output
frequency if it operates at 35 MHz? ∴ The operating frequency is
100.998485 MHz
Notations:
PROBLEM 63
fo = output frequency (Hz)
Varactor modulators directly change the
f = operating frequency (Hz) frequency of a carrier signal proportional to
the amplitude of the modulating signal. If the
k = accuracy
sensitivity of the modulator kf = 3kHz/V and
∆f = change in frequency (Hz) the maximum deviation is 75 kHz and is
followed by 2 tripler and 1 doubler, what
Solution:
audio voltage is needed at the modulator to
𝑓𝑜 = 𝑓 + ∆𝑓 obtain full deviation?

∆𝑓 = 𝑘𝑓 Notations:
0.001
𝑓 = 35 𝑀𝐻𝑧 + (35 × 106 𝐻𝑧) ( 100 ) δout = output of multiplier circuits (Hz)

δosc = output of oscillator circuits (Hz)


𝑓 = 35.00035 𝑀𝐻𝑧
Vi = peak audio voltage (V)
∴ The operating frequency is 35.00035
MHz. V = audio voltage RMS (V)
PROBLEM 62 Solution:
An FM radio transmitter transmits at exactly 𝛿𝑜𝑢𝑡 = (𝛿𝑜𝑠𝑐 )(3)(3)(2)
101 MHz at 20°C. The transmitter uses
𝛿𝑜𝑢𝑡 = 75 𝑘𝐻𝑧
crystal oscillator with temperature
coefficient of -1 ppm/°C. Find the 𝛿𝑜𝑠𝑐 =
𝛿𝑜𝑢𝑡
=
75 𝑀𝐻𝑧
18 18
𝑉𝑖 =
𝛿𝑜𝑠𝑐 Co = capacitance at zero bias voltage (F)
𝑘𝑓

4167 𝐻𝑧
Solution:
𝑉𝑖 = 3000 𝐻𝑧/𝑉 = 1.389𝑉
1
𝑓𝑜
2𝜋√𝐿𝐶
𝑉𝑖 1.389
𝑉= = = 0.982 𝑉 = 982 𝑚𝑉
√2 √2 1
𝐶 = 4𝜋2 𝑓 2 𝐿
𝑜
∴ The rms audio voltage is 982 mV.
Since frequency is increased by 75%
PROBLEM 64
𝑓𝑜 = 14.35 × 106
A varactor is used to tune the receiver at 1.78
1
MHz, with 150 uH. What is the maximum 𝐶 = 4𝜋2 (14.35×106 )2 (75×10−6 )
capacitance if no tuning voltage is applied?
𝐶 = 1.64 𝑝𝐹
Notations:
𝐶𝑜
𝐶=
fo = operating frequency (Hz) √1+2𝑉

𝐶 2
L = inductance (H) ( 𝑜) −1
𝐶
𝑉= 2
C = capacitance (F)
5 2
( ) −1
1.64
Solution: 𝑉=
2
= 4.15 𝑉

The maximum capacitance of the varactor where:


occurs for zero bias voltage.
𝐶 = 1.64 𝑝𝐹
1
𝐶=
(2𝜋𝑓𝑜 )2 𝐿 𝐶𝑜 = 5 𝑝𝐹
1
𝐶 = [2𝜋(1.78×106 )]2 (150×10−6 ) = 53.3 𝑝𝐹 ∴ The bias voltage at the varactor is
4.14 V.
∴ The maximum capacitance is 53.3 pF.
PROBLEM 66
PROBLEM 65
A PLL has a VCO with a capture range of 6
A tuned circuit resonates at a frequency of MHz and a lock range of 14 MHz. Determine
8.2 MHz. A varactor diode is used that has a the free running frequency if the input is
maximum capacitance of 5 pF with a 75 uH gradually raised from zero then the loop
inductor when tuning voltage is zero. What is locks at 10 MHz.
the tuning voltage if the input frequency is
increased by 75%? Notations:

Notations: fFR = free-running frequency (Hz)

fo = operating frequency (Hz) fLR = lock range (Hz)

L = inductance (H) fCR = capture range (Hz)

C = capacitance (F) fi = input frequency lock is first achieved


(Hz)
V = bias voltage (V)
Solution: The way frequency at which lock is lost on
the way up
𝑓𝐶𝑅 = 2(𝑓𝐹𝑅 − 𝑓𝑖 )
𝑓𝐿𝑅
𝑓𝐶𝑅 𝑓𝑖𝐿 = 𝑓𝐹𝑅 +
𝑓𝑜 = 2
+ 𝑓𝑖 2
8 𝑀𝐻𝑧
6 𝑀𝐻𝑧 𝑓𝑖𝐿 = 14 𝑀𝐻𝑧 +
𝑓𝑜 = + 10 𝑀𝐻𝑧 2
2
∴ The frequency at which lock is
𝑓𝑜 = 13 𝑀𝐻𝑧
achieved is 14 MHz and is lost at 18
∴ The free running frequency is 13 MHz. MHz.

PROBLEM 67 PROBLEM 68

Suppose that the free running frequency of An amplifier is operating at class C with an
the VCO is 14 MHz, capture range 4 MHz, and efficiency of 80%. What power will it deliver
lock range 8 MHz. What would be the to the load if the power drawn from the
frequencies at which lock is achieved and lost source is 125 W?
provided the reference input frequency
Notations:
begins at zero?
Pout = output power (W)
Notations:
Pin = input power (W)
fFR = free-running frequency (Hz)
η = efficiency
fLR = lock range (Hz)
Solution:
fCR = capture range (Hz)
𝑃𝑜𝑢𝑡 = 𝜂𝑃𝑖𝑛
fiL = input frequency lock is lost (Hz)
𝑃𝑜𝑢𝑡 = (0.8)(125)
fiC = input frequency lock is first achieved
(Hz) 𝑃𝑜𝑢𝑡 = 100 𝑊

Solution: ∴ The output power is 100 W.


𝑓𝐶𝑅 = 2(𝑓𝐹𝑅 ± 𝑓𝑖𝐶 ) PROBLEM 69
𝑓𝐿𝑅 = 2(𝑓𝐹𝑅 ± 𝑓𝑖𝐿 ) A BJT transistor has a power dissipation
𝑓𝐶𝑅 rating of 15 W. Determine the power an
𝑓𝑖𝐶 = 𝑓𝐹𝑅 ± 2 amplifier could deliver to the load operating
at class A with an efficiency of 30%. Suppose
Frequency at which lock is first achieved
that the transistor is the only element that
from zero frequency
dissipates power in the circuit.
𝑓𝐶𝑅
𝑓𝑖𝐶 = 𝑓𝐹𝑅 − Notations:
2
4 𝑀𝐻𝑧
𝑓𝑖𝐶 = 14 𝑀𝐻𝑧 − = 12 𝑀𝐻𝑧 Pout = output power (W)
2
𝑓𝐶𝑅 Pin = input power (W)
𝑓𝑖𝐿 = 𝑓𝐹𝑅 ± 2
PDis = power dissipated (W)
η = efficiency the situation above if the inductance of 25
uH is used?
Solution:
Notations:
𝑃𝑖𝑛 = 𝑃𝑜𝑢𝑡 + 𝑃𝐷𝑖𝑠
fo = operating frequency (Hz)
𝑃𝐷𝑖𝑠 = 𝑃𝑖𝑛 − 𝑃𝑜𝑢𝑡 Eq. (1)
𝑃𝑜𝑢𝑡 L = inductance (H)
𝑃𝑖𝑛 = 𝜂
Eq. (2)
C = capacitance (F)
Eq. (2) in Eq. (1)
V = bias voltage (V)
𝑃𝑜𝑢𝑡
𝑃𝐷𝑖𝑠 = − 𝑃𝑜𝑢𝑡
𝜂 Co = capacitance at zero bias voltage (F)

𝑃𝐷𝑖𝑠 = 𝑃𝑜𝑢𝑡 (𝜂 − 1)
1
Solution:
1
15 𝑓 = 2𝜋
𝑃𝑜𝑢𝑡 = 1 = 6.43 𝑊 √𝐿𝐶
( −1)
0.3
1
𝐶10𝑉 = (2𝜋𝑓)2 𝐿
∴ The power that can be delivered to
the load is 6.43 W. 1
𝐶10𝑉 = [2𝜋(15×108 ]2 (25×10−6 )
PROBLEM 70
𝐶10𝑉 = 4.5 𝑝𝐹
A local oscillator uses a frequency 𝐶𝑜
multiplication circuit consisting of two triplers 𝐶=
√1+2𝑉
and three doublers to get a frequency of 654
MHz from a crystal oscillator. What frequency 𝐶𝑜 = 𝐶10 (√1 + 2𝑉)
should the crystal oscillator operate?
𝐶𝑜 = 4 × 10−12 (√1 + 2(10)) = 20.6 𝑝𝐹
Notations:
∴ The capacitance at zero bias is 20.6 pF.
fout = output frequency of the multiplier (Hz)
PROBLEM 72
fosc = output frequency of the oscillator (Hz)
A PLL has a free running frequency of 12
Solution: MHz with a capture range of 2 MHz and lock
range of 4 MHz. What is the lowest input
𝑓𝑜𝑢𝑡 = (𝑓𝑜𝑠𝑐 )(3)(3)(2)(2)(2) = 654 𝑀𝐻𝑧
frequency of the PLL that will cause the
𝑓𝑜𝑠𝑐 =
𝑓𝑜𝑢𝑡
=
654 𝑀𝐻𝑧
= 9.08 𝑀𝐻𝑧 unlocked loop to lock?
72 72
Notations:
∴ The output frequency of the oscillator
is 9.08 MHz. fFR = free-running frequency (Hz)
PROBLEM 71 fLR = lock range (Hz)
The resonant frequency of a varactor-tuned fCR = capture range (Hz)
oscillator is 15 MHz when the tuning voltage
fi = input frequency lock is first achieved
of the varactor reaches 10 V. What is the
(Hz)
maximum capacitance of the varactor given
Solution: fi = input frequency lock is first achieved
𝑓𝐶𝑅 (Hz)
𝑓𝑖𝐿 = 𝑓𝐹𝑅 ± 2
Solution:
Lowest input frequency lock is first achieved 𝑓𝐿𝑅
𝑓𝑖𝐿 = 𝑓𝐹𝑅 ± 2
2 𝑀𝐻𝑧
𝑓𝑖𝐿 = 12 𝑀𝐻𝑧 − 2
Highest input frequency lock is lost
𝑓𝑖𝐿 = 11 𝑀𝐻𝑧 𝑓𝐿𝑅
𝑓𝑖𝐿 = 𝑓𝐹𝑅 + 2
∴ The lowest input frequency is 11 MHz.
2 𝑀𝐻𝑧
PROBLEM 73 𝑓𝑖𝐿 = 10 𝑀𝐻𝑧 + 2

A frequency-modulation system has a 𝑓𝑖𝐿 = 11 𝑀𝐻𝑧


maximum allowable modulating voltage of
∴ The highest input frequency is 11
10V. If the sensitivity of the modulator is 2
MHz.
kHz/V, what is the system frequency
deviation? PROBLEM 75

Notation: An FM system has a rated system deviation


of 15 kHz and a maximum allowable
δ = frequency deviation (Hz)
modulating voltage of 5 V. Calculate the
Vm = peak amplitude of the modulating sensitivity of the modulator.
signal (V)
Notation:
K = sensitivity (Hz/V)
δ = frequency deviation (Hz)
Solution:
Vm = peak amplitude of the modulating
𝛿 = 𝑘𝑉𝑚 signal (V)

𝛿 = (2 𝑘𝐻𝑧/𝑉) × 10 𝑉 = 20 𝑘𝐻𝑧 K = sensitivity (Hz/V)

∴ The frequency deviation is 20 kHz. Solution:

PROBLEM 74 𝛿 = 𝑘𝑉𝑚
𝛿
A PLL has a free running frequency of 10 MHz 𝑘=𝑉
𝑚
with a capture range of 1 MHz and lock range
15 𝑘𝐻𝑧
of 2 MHz. What is the highest input 𝑘= = 3 𝑘𝐻𝑧/𝑉
5𝑉
frequency of the PLL that can be tracked by
lock loop? ∴ The sensitivity of the modulator is 3
kHz/V.
Notations:
PROBLEM 76
fFR = free-running frequency (Hz)
An FM modulator has sensitivity equal to 2
fLR = lock range (Hz)
kHz/V and a rated system deviation of 3 kHz.
fCR = capture range (Hz) What is the maximum allowable signal
voltage?
Notations: PROBLEM 78

𝛿 = 𝑓𝑟𝑒𝑞𝑢𝑒𝑛𝑐𝑦 𝑑𝑒𝑣𝑖𝑎𝑡𝑖𝑜𝑛 An FM linear modulator operates at 92 MHz


(carrier frequency). It is modulated by a 5
𝑉𝑚 =
kHz sinusoidal signal. Calculate the
𝑝𝑒𝑎𝑘 𝑎𝑚𝑝𝑙𝑖𝑡𝑢𝑑𝑒 𝑜𝑓 𝑡ℎ𝑒 𝑚𝑜𝑑𝑢𝑙𝑎𝑡𝑖𝑜𝑛𝑔 𝑠𝑖𝑔𝑛𝑎𝑙 (𝑉)
modulation index if the instantaneous carrier
𝐻𝑧
𝑘 = 𝑠𝑒𝑛𝑠𝑖𝑡𝑖𝑣𝑖𝑡𝑦 ( 𝑉 ) frequency varies between 91.975 and 92.025
MHz.
Solution:
Notations:
𝛿 = 𝑘𝑉𝑚
𝛿 = 𝑓𝑟𝑒𝑞𝑢𝑒𝑛𝑐𝑦 𝑑𝑒𝑣𝑖𝑎𝑡𝑖𝑜𝑛 (𝐻𝑧)
𝛿
𝑉𝑚 = 𝑘 𝑚𝑓 = 𝐹𝑀 𝑚𝑜𝑑𝑢𝑙𝑎𝑡𝑖𝑜𝑛 𝑖𝑛𝑑𝑒𝑥
3 𝑘𝐻𝑧
𝑉𝑚 = 2 𝑘𝐻𝑧/𝑉 = 1.5 𝑉 𝑓𝑚 = 𝑚𝑜𝑑𝑢𝑙𝑎𝑡𝑖𝑜𝑛𝑔 𝑓𝑟𝑒𝑞𝑢𝑒𝑛𝑐𝑦 (𝐻𝑧)

⸫ The maximum allowable signal 𝐶𝑆 = 𝑐𝑎𝑟𝑟𝑖𝑒𝑟 𝑠𝑤𝑖𝑛𝑔 (𝐻𝑧)


voltage is 1.5 V. Solution:
PROBLEM 77 𝐶𝑆 = (92.025 − 91.975) 𝑀𝐻𝑧
An FM modulator has sensitivity equal to 2 𝐶𝑆 = 0.05 𝑀𝐻𝑧
kHz/V and a rated system deviation of 3 kHz.
𝐶𝑆 50𝑘𝐻𝑧
Calculate the carrier swing if a 0.5 V signal is 𝛿= 2
= 2
= 25 𝑘𝐻𝑧
applied to the modulator?
𝐶𝑆 = 50 𝑘𝐻𝑧
Notations:
𝛿
𝑚𝑓 = 𝑓
𝛿 = 𝑓𝑟𝑒𝑞𝑢𝑒𝑛𝑐𝑦 𝑑𝑒𝑣𝑖𝑎𝑡𝑖𝑜𝑛 𝑚

25
𝑉𝑚 = 𝑚𝑓 = 5
=5
𝑝𝑒𝑎𝑘 𝑎𝑚𝑝𝑙𝑖𝑡𝑢𝑑𝑒 𝑜𝑓 𝑡ℎ𝑒 𝑚𝑜𝑑𝑢𝑙𝑎𝑡𝑖𝑜𝑛𝑔 𝑠𝑖𝑔𝑛𝑎𝑙 (𝑉)
⸫ The modulating index is 5.
𝐻𝑧
𝑘 = 𝑠𝑒𝑛𝑠𝑖𝑡𝑖𝑣𝑖𝑡𝑦 (𝑉)
PROBLEM 79
𝐶𝑆 = 𝑐𝑎𝑟𝑟𝑖𝑒𝑟 𝑠𝑤𝑖𝑛𝑔 (𝐻𝑧) An FM linear modulator has sensitivity equal
Solution: to 2 kHz/V and maximum allowable
modulating signal voltage of 5 V. If the
𝛿 = 𝑘𝑉𝑚 instantaneous carrier frequency varies
𝑘𝐻𝑧 between 98.95 and 99.05 MHz, calculate the
𝛿 = (2 )𝑥 0.5 𝑉 = 1𝑘𝐻𝑧
𝑉 percent modulation.
𝐶𝑆 = 2 𝑥 𝛿
Notations:
𝐶𝑆 = 2 𝑥 1 𝑘𝐻𝑧 = 2𝑘𝐻𝑧 𝛿𝑎𝑐𝑡 = 𝑎𝑐𝑡𝑢𝑎𝑙 𝑓𝑟𝑒𝑞𝑢𝑒𝑛𝑐𝑦 𝑑𝑒𝑣𝑖𝑎𝑡𝑖𝑜𝑛 (𝐻𝑧)
⸫ The carrier swing of the signal is 𝛿𝑚𝑎𝑥 =
2kHz. 𝑚𝑎𝑥𝑖𝑚𝑢𝑚 𝑓𝑟𝑒𝑞𝑢𝑒𝑛𝑐𝑦 𝑑𝑒𝑣𝑖𝑎𝑡𝑖𝑜𝑛 (𝐻𝑧)
𝑉𝑚 = 𝑓𝑚 = 𝑚𝑜𝑑𝑢𝑙𝑎𝑡𝑖𝑜𝑛𝑔 𝑓𝑟𝑒𝑞𝑢𝑒𝑛𝑐𝑦 (𝐻𝑧)
𝑝𝑒𝑎𝑘 𝑎𝑚𝑝𝑙𝑖𝑡𝑢𝑑𝑒 𝑖𝑓 𝑡ℎ𝑒 𝑚𝑜𝑑𝑢𝑙𝑎𝑡𝑖𝑛𝑔 𝑠𝑖𝑔𝑛𝑎𝑙 (𝑉)
Solution:
𝑘 = 𝑠𝑒𝑛𝑠𝑖𝑡𝑖𝑣𝑖𝑡𝑦 (𝐻𝑧/𝑉)
𝑉𝑑𝐵 = 20 log 𝑉𝑆 /𝑉𝑁
𝐶𝑆 = 𝑐𝑎𝑟𝑟𝑖𝑒𝑟 𝑠𝑤𝑖𝑛𝑔 (𝐻𝑧) 𝑉𝑆 𝑉𝑑𝐵
𝑉𝑁
= log −1 20
= 31.62
Solution:
𝑉 𝑉𝑁
𝛿𝑎𝑐𝑡 = 𝑘𝑉𝑚 𝛷𝑚𝑎𝑥 = sin−1 𝑉𝑁 = 𝑉𝑆
𝑆

𝑘𝐻𝑧 1
𝛿𝑎𝑐𝑡 = (2 )𝑥 5𝑉 𝛷𝑚𝑎𝑥 = 31.62
𝑉

𝛿𝑎𝑐𝑡 = 10 𝑘𝐻𝑧 𝛿𝑁 = 𝛷𝑓𝑚 = 𝑚𝑓 𝑓𝑚


𝐶𝑆
𝛿𝑚𝑎𝑥 = 𝛿𝑁 = 𝛷𝑚𝑎𝑥 𝑥 3 𝑘𝐻𝑧 = 95 𝐻𝑧
2

99.05−98.95 𝛿𝑆 = 𝑚𝑓 𝑥 3 𝑘𝐻𝑧 = 1500 𝐻𝑧


𝛿𝑚𝑎𝑥 = ( 2
) 𝑀𝐻𝑧
𝑆 𝛿𝑆
= 20 log
𝛿𝑚𝑎𝑥 = 0.05 𝑀𝐻𝑧 𝑁 𝛿𝑁

𝛿 𝑆 1500
%𝑀 = 𝛿 𝑎𝑐𝑡 𝑥 100% 𝑁
= 20 log 95
= 24 𝑑𝐵
𝑚𝑎𝑥
⸫ The output signal-to-noise-ratio of
10 𝑘𝐻𝑧
%𝑀 = 50 𝑘𝐻𝑧 𝑥 100% the detector is 24 dB.

%𝑀 = 20% PROBLEM 81

⸫ The percent modulating is 20%. Initially, FM signal bandwidth is 100 kHz


when the modulation index is 5. What is the
PROBLEM 80 new modulation index if the signal level is
An FM signal has a maximum modulating increased by 8 dB?
frequency of 3 kHz and a modulation index Notations:
of 0.5. The signal-to-noise ratio at the input
of the detector is 30 dB. Calculate the 𝑉 = 𝑠𝑖𝑔𝑛𝑎𝑙 𝑣𝑜𝑙𝑡𝑎𝑔𝑒 (𝑉)
approximate signal-to-noise ratio at the 𝑚𝑓 = 𝐹𝑀 𝑚𝑜𝑑𝑢𝑙𝑎𝑡𝑖𝑜𝑛 𝑖𝑛𝑑𝑒𝑥
detector’s output.
𝐻𝑧
𝑘 = 𝑠𝑒𝑛𝑠𝑖𝑡𝑖𝑣𝑖𝑡𝑦 ( 𝑉 )
Notations:
𝛷𝑁 = 𝑝ℎ𝑎𝑠𝑒 𝑑𝑒𝑣𝑖𝑎𝑡𝑖𝑜𝑛 𝑑𝑢𝑠𝑒 𝑡𝑜 𝑛𝑜𝑖𝑠𝑒 (𝑟𝑎𝑑) 𝑓𝐶 = 𝑐𝑎𝑟𝑟𝑖𝑒𝑟 𝑓𝑟𝑒𝑞𝑢𝑒𝑛𝑐𝑦 (𝐻𝑧)

𝛷𝑆 = 𝑝ℎ𝑎𝑠𝑒 𝑑𝑒𝑣𝑖𝑎𝑡𝑖𝑜𝑛 𝑑𝑢𝑠𝑒 𝑡𝑜 𝑠𝑖𝑔𝑛𝑎𝑙 (𝑟𝑎𝑑) 𝑓𝑚 = 𝑚𝑜𝑑𝑢𝑙𝑎𝑡𝑖𝑜𝑛𝑔 𝑓𝑟𝑒𝑞𝑢𝑒𝑛𝑐𝑦 (𝐻𝑧)

𝑉𝑁 = 𝑛𝑜𝑖𝑠𝑒 𝑣𝑜𝑙𝑡𝑎𝑔𝑒 (𝑉) Solution:


𝑉𝑛𝑒𝑤
𝑉𝑆 = 𝑠𝑖𝑔𝑛𝑎𝑙 𝑣𝑜𝑙𝑡𝑎𝑔𝑒 (𝑉) 𝑉𝑑𝐵 = 20 log
𝑉𝑜𝑙𝑑
𝑆 𝑉𝑛𝑒𝑤 𝑉𝑑𝐵
𝑁
= 𝑠𝑖𝑔𝑛𝑎𝑙 − 𝑡𝑜 − 𝑛𝑜𝑖𝑠𝑒 𝑝𝑜𝑤𝑒𝑟 𝑟𝑎𝑡𝑖𝑜 = log −1
𝑉𝑜𝑙𝑑 20
𝑉𝑛𝑒𝑤
= 2.5 dB, when the maximum modulating
𝑉𝑜𝑙𝑑
frequency is 15 kHz and the frequency
𝑚𝑓 =
𝑘𝑉𝑚 𝑓𝑐
deviation is 75 kHz. What will be the new
𝑓𝑚
output signal-to-noise ratio if the deviation
𝑠𝑖𝑚𝑐𝑒 𝑚𝑓 𝑖𝑠 𝑑𝑖𝑟𝑒𝑐𝑡𝑙𝑦 𝑝𝑟𝑜𝑝𝑜𝑟𝑡𝑖𝑜𝑛𝑎𝑙 𝑡𝑜 𝑉𝑚 𝑠𝑜, ratio is increased to 8?

𝑚𝑓(𝑛𝑒𝑤) = 2.5[𝑚𝑓(𝑜𝑙𝑑) ] Notations:

𝑚𝑓(𝑛𝑒𝑤) = 2.5[5] = 12.5 𝐷𝑅 = 𝑑𝑒𝑣𝑖𝑎𝑡𝑖𝑜𝑛 𝑟𝑎𝑡𝑖𝑜


𝑆
⸫ The new modulation index is 12.5. 𝑁
= 𝑠𝑖𝑔𝑛𝑎𝑙 − 𝑡𝑜 − 𝑛𝑜𝑖𝑠𝑒 𝑝𝑜𝑤𝑒𝑟 𝑟𝑎𝑡𝑖𝑜

PROBLEM 82 𝛿 = 𝑓𝑟𝑒𝑞𝑢𝑒𝑛𝑐𝑦 𝑑𝑒𝑣𝑖𝑎𝑡𝑖𝑜𝑛 (𝐻𝑧)


An FM broadcast transmitter has a bandwidth 𝑓𝑚(max) =
of 180 kHz and deviation ratio of 10. Suppose 𝑚𝑎𝑥𝑖𝑚𝑢𝑚 𝑚𝑜𝑑𝑢𝑙𝑎𝑡𝑖𝑜𝑛𝑔 𝑓𝑟𝑒𝑞𝑢𝑒𝑛𝑐𝑦 (𝐻𝑧)
the carrier frequency is 98.2 MHz, what is the
maximum allowable modulating frequency? Solution:
𝛿
Notations: 𝐷𝑅𝑜𝑙𝑑 =
𝑓𝑚𝑎𝑥

𝐵𝑊 = 𝐵𝑎𝑛𝑑𝑤𝑖𝑑𝑡ℎ (𝐻𝑧) 𝐷𝑅𝑜𝑙𝑑 = 15 = 5


75

𝛿 = 𝑓𝑟𝑒𝑞𝑢𝑒𝑛𝑐𝑦 𝑑𝑒𝑣𝑖𝑎𝑡𝑖𝑜𝑛 (𝐻𝑧) 𝑆 𝑆 𝐷𝑅𝑛𝑒𝑤


( ) =( ) + 20 log
𝑁 𝑛𝑒𝑤 𝑁 𝑜𝑙𝑑 𝐷𝑅𝑜𝑙𝑑
𝑓𝑚(max) =
𝑚𝑎𝑥𝑖𝑚𝑢𝑚 𝑚𝑜𝑑𝑢𝑙𝑎𝑡𝑖𝑜𝑛𝑔 𝑓𝑟𝑒𝑞𝑢𝑒𝑛𝑐𝑦 (𝐻𝑧) (𝑁)
𝑆
= 30 𝑑𝐵 + 20 log 5
8
𝑛𝑒𝑤
𝐷𝑅 = 𝑑𝑒𝑣𝑖𝑎𝑡𝑖𝑜𝑛 𝑟𝑎𝑡𝑖𝑜
𝑆
( ) = 30 𝑑𝐵 + 4.1 𝑑𝐵
𝑁 𝑛𝑒𝑤
Solution:
𝑆
𝐵𝑊 = 2(𝛿 + 𝑓𝑚𝑎𝑥 ) ( ) = 34.1 𝑑𝐵
𝑁 𝑛𝑒𝑤
𝛿
𝐷𝑅 = 𝑓 ⸫ The new signal-to-noise ratio is 34.1
𝑚𝑎𝑥
dB.
𝑇ℎ𝑢𝑠:
PROBLEM 84
𝐵𝑊
( )
2
𝑓𝑚𝑎𝑥 = 1+𝐷𝑅 An FM receiver tuned at 91.9 MHz uses two
180 𝑘𝐻𝑧 unoccupied tuned circuits before the mixer,
( ) 90
𝑓𝑚𝑎𝑥 = 2
= 11 = 8.2 𝑘𝐻𝑧 each with a Q of 65. If the receiver’s IF is
1+10
10.7 MHz and the local oscillator uses low
⸫ The maximum modulating frequency side injection, what is the image rejection
us 8.2 kHz. ratio of the receiver in dB?
PROBLEM 83 Notations:
An FM system has a rated bandwidth of 180 𝐼𝑅 = 𝑖𝑚𝑎𝑔𝑒 𝑓𝑟𝑒𝑞𝑢𝑒𝑛𝑐𝑦 𝑟𝑒𝑗𝑒𝑐𝑡𝑖𝑜𝑛 𝑟𝑎𝑡𝑖𝑜
kHz and an output signal-to-noise ratio of 30
𝑓𝑠 = 𝑠𝑖𝑔𝑛𝑎𝑙 𝑓𝑟𝑒𝑞𝑢𝑒𝑛𝑐𝑦 (𝐻𝑧) 𝑅𝑉 = 𝑣𝑒𝑟𝑡𝑖𝑐𝑎𝑙 𝑟𝑒𝑠𝑜𝑙𝑢𝑡𝑖𝑜𝑛

𝑓𝑠𝑖 = 𝑖𝑚𝑎𝑔𝑒 𝑓𝑟𝑒𝑞𝑢𝑒𝑛𝑐𝑦 (𝐻𝑧) 𝑁𝐿 = 𝑎𝑐𝑡𝑢𝑎𝑙 𝑙𝑖𝑛𝑒𝑠 𝑠ℎ𝑜𝑤𝑛

𝑓𝑖 = 𝑖𝑛𝑡𝑒𝑟𝑚𝑒𝑑𝑖𝑎𝑡𝑒 𝑓𝑟𝑒𝑞𝑢𝑒𝑛𝑐𝑦 (𝐻𝑧) Solution:

𝑄 = 𝑞𝑢𝑎𝑙𝑖𝑡𝑦 𝑓𝑎𝑐𝑡𝑜𝑟 𝑅𝑉 = 0.7𝑁𝐿

Solution: 𝑅𝑉 = 0.7(485)

𝐼𝑅 = √1 + 𝑄 2 𝑝2 𝑅𝑉 = 340 𝑙𝑖𝑛𝑒𝑠

𝐼𝑅 = √1 + (65)2 (0.54)2 ⸫ The vertical resolutions are 340 lines.

𝐼𝑅 = 35.1 PROBLEM 2

where: Past ECE Board Problem

𝑓𝑠𝑖 𝑓𝑠 Calculate the video-frequency response


𝜌=| − |
𝑓𝑠 𝑓𝑠𝑖 corresponding to the horizontal resolution of
70.5 91.9 200 lines of a TV receiver.
𝜌 = |91.9 − 70.5|
Notations:
𝑓𝑠𝑖 = 𝑓𝑠 − 2𝑓𝑖
𝑓 = 𝑣𝑖𝑑𝑒𝑜 − 𝑓𝑟𝑒𝑞𝑢𝑒𝑛𝑐𝑦 𝑟𝑒𝑠𝑝𝑜𝑛𝑠𝑒
𝑓𝑠𝑖 = 91.9 − 2(10.7)
𝑇𝐻 = ℎ𝑜𝑟𝑖𝑧𝑜𝑛𝑡𝑎𝑙 𝑙𝑖𝑛𝑒 𝑠𝑦𝑛𝑐ℎ𝑟𝑜𝑛𝑖𝑧𝑎𝑡𝑖𝑜𝑛 𝑡𝑖𝑚𝑒
𝑓𝑠𝑖 = 70.5 𝑀𝐻𝑧
= 63.5 µ𝑠 (𝑁𝑇𝑆𝐶 𝑠𝑡𝑎𝑛𝑑𝑎𝑟𝑑)
For two uncoupled tuned circuit
𝑇𝑆 = 𝑙𝑖𝑛𝑒 𝑠𝑢𝑠𝑝𝑒𝑛𝑠𝑖𝑜𝑛 𝑝𝑒𝑟𝑖𝑜𝑑
𝐼𝑅𝑇 = (𝐼𝑅)2
= 10 µ𝑠 (𝑁𝑇𝑆𝐶 𝑠𝑡𝑎𝑛𝑑𝑎𝑟𝑑)
𝐼𝑅𝑇𝑑𝐵 = 20 log(𝐼𝑅)2
Solution:
𝐼𝑅𝑇𝑑𝐵 = 20 log(35.1)2
𝑁ℎ𝑜𝑟 1
𝑓= 2
(𝑇 −𝑇 )
𝐼𝑅𝑇𝑑𝐵 = 61.8 𝑑𝐵 𝐻 𝑆

200 1
⸫ The image frequency rejection ratio 𝑓= (
2 63.5𝑥10−6 −10𝑥10−6
)
is 61.8 dB.
𝑓 = 1.87 𝑀𝐻𝑧
Question Bank 7: TELEVISION AND
⸫ The video-frequency response is 1.87
FACSIMILE
MHz.
PROBLEM 1
PROBLEM 3
In TV, the vertical resolution is the number
Past ECE Board Problem
of horizontal lines that can be distinguished.
If 485 lines are shown on the screen, In facsimile transmission, of the drum
determine the vertical resolution. diameter is 60mm and the scanning pitch is
0.1 mm, calculate the index of cooperation
Notations:
using IEEE recommendation.
Notations: Notations:
𝐼𝑂𝐶 = 𝑖𝑛𝑑𝑒𝑥 𝑜𝑓 𝑐𝑜𝑜𝑝𝑒𝑟𝑎𝑡𝑖𝑜𝑛 𝑇𝐻 = ℎ𝑜𝑟𝑖𝑧𝑜𝑛𝑡𝑎𝑙 𝑙𝑖𝑛𝑒 𝑠𝑦𝑛𝑐ℎ𝑟𝑜𝑛𝑖𝑧𝑎𝑡𝑖𝑜𝑛 𝑡𝑖𝑚𝑒

𝐷 = 𝑑𝑟𝑢𝑚 𝑑𝑖𝑎𝑚𝑒𝑡𝑒𝑟 (𝑚𝑚) = 63.5 µ𝑠 (𝑁𝑇𝑆𝐶 𝑠𝑡𝑎𝑛𝑑𝑎𝑟𝑑)

𝑃 = 𝑠𝑐𝑎𝑛𝑛𝑖𝑛𝑔 𝑝𝑖𝑡𝑐ℎ (𝑚𝑚) 𝑇𝑆 = 𝑙𝑖𝑛𝑒 𝑠𝑢𝑠𝑝𝑒𝑛𝑠𝑖𝑜𝑛 𝑝𝑒𝑟𝑖𝑜𝑑

Solution: = 10 µ𝑠 (𝑁𝑇𝑆𝐶 𝑠𝑡𝑎𝑛𝑑𝑎𝑟𝑑)

𝐹𝑜𝑟 𝐼𝐸𝐸𝐸, 𝑓 = ℎ𝑖𝑔ℎ𝑒𝑠𝑡 𝑣𝑖𝑑𝑒𝑜 𝑓𝑟𝑒𝑞𝑢𝑒𝑛𝑐𝑦


𝜋𝐷 𝜋(60 𝑚𝑚) 𝑎 = 𝑎𝑠𝑝𝑒𝑐𝑡 𝑟𝑎𝑡𝑖𝑜
𝐼𝑂𝐶 = 𝑃
= 0.1 𝑚𝑚
𝑁 = ℎ𝑜𝑟𝑖𝑧𝑜𝑛𝑡𝑎𝑙 𝑟𝑒𝑠𝑜𝑙𝑢𝑡𝑖𝑜𝑛
𝐼𝑂𝐶 = 1885
𝑁𝑠 = 𝑠𝑢𝑝𝑝𝑟𝑒𝑠𝑠𝑒𝑑 ℎ𝑜𝑟𝑖𝑧𝑜𝑛𝑡𝑎𝑙 𝑙𝑖𝑛𝑒𝑠
⸫ The index of cooperation using IEEE
recommendations is 1885. Solution:

PROBLEM 4 In practice, the highest video frequency is


standardized at
Past ECE Board Problem
𝑁−𝑁𝑆
A signal in a cable television system has an 𝑓 = 0.35𝑎 ( ) , 𝑀𝐻𝑧
𝑇𝐻 (µ𝑠)−𝑇𝑆 (µ𝑠)
amplitude of 4 mV in a 75-ohm resistive line. 320−40
Calculate its level in dBm. 𝑓 = 0.35(1.33) (63.5−10)

Notations: 𝑓 = 2.4 𝑀𝐻𝑧

𝑑𝐵𝑚 = 𝑑𝐵 𝑟𝑒𝑓𝑒𝑟𝑟𝑒𝑑 𝑡𝑜 1 𝑚𝑖𝑙𝑙𝑖𝑤𝑎𝑡𝑡 where:

Solution: 𝑎 = 𝑎𝑠𝑝𝑒𝑐𝑡 𝑟𝑎𝑡𝑖𝑜 = 1.33


𝑃 𝑇𝐻 = 63.5 µ𝑠
𝑑𝐵𝑚 = 10 log( )
1𝑥10−3

0.213µ𝑊 𝑇𝑆 = µ𝑠
𝑑𝐵𝑚 = 10 log( )
1𝑥10−3
𝑁𝑆 = 40 𝑙𝑖𝑛𝑒𝑠
𝑑𝐵𝑚 = −36.7 𝑑𝐵𝑚
Note:
𝑊ℎ𝑒𝑟𝑒:
Since the lowest frequency in the video signal
2
𝑃=
(4𝑥10−3 )
= 0.213 µ𝑊 is nearly 0, therefore the highest video
75
frequency also gives the video bandwidth
⸫ The signal level is -36.7 dBm. required.

PROBLEM 5 ⸫ The video-frequency response is 2.4


MHz.
Past ECE Board Problem

Calculate the video-frequency response


corresponding to the horizontal resolution of
320 lines of a TV receiver.
PROBLEM 6 ∴ The maximum bandwidth of the
signal that the TV can handle is 6.2MHz.
Past ECE Board Problem
Question Bank 8: MICROWAVE
For a TV system, the horizontal flyback is 10
COMMUNICATIONS
percent, this time equals ____.
PROBLEM 1
Notations:
Notations:
𝑇𝐻 = ℎ𝑜𝑟𝑖𝑧𝑜𝑛𝑡𝑎𝑙 𝑙𝑖𝑛𝑒 𝑠𝑦𝑛𝑐ℎ𝑟𝑜𝑛𝑖𝑧𝑎𝑡𝑖𝑜𝑛 𝑡𝑖𝑚𝑒
Lp = free space path loss or attenuation
= 63.5 µ𝑠 (𝑁𝑇𝑆𝐶 𝑠𝑡𝑎𝑛𝑑𝑎𝑟𝑑)
(dB)
Solution:
D = path distance (km)
10%(𝑇𝐻 ) = 10%(63.5µ𝑠) = 6.4µ𝑠
F = frequency (GHz)
⸫ The horizontal flyback time is 6.4 µs.
Solution:
PROBLEM 7
Lp= 96.6 + 20logdmi + 20logfGHz
For analog TV transmission, typically the
Lp = 96.6 + 20log (11,000) + 20log1.2
bandwidth of video signal is up to 4 MHz. If
the vertical resolution is increased to 500 Lp = 179. dB
lines, determine the maximum bandwidth of
∴ The path attenuation is 179 dB.
the signal that the TV can handle.
PROBLEM 2
Solution:

Using 32 line for vertical trace Calculate the beamwidth of a fully


illuminated microwave passive repeater
Horizontal line = (500/0.7) + 32 = 747 lines whose effective linear dimension is 25 feet
with an operating frequency of 6GHz.
A total of 30 frames scan in one second,
therefore Notations:
# of horizontal line = frame time / line time ϴ = beamwidth (degrees)
1/30
line time = = 44.02 µs 𝜆 = signal wavelength (ft)
747

If 10µs is used for horizontal blanking pulse, L = linear dimension (ft)


therefore Solution:
Trace time = 44.62 – 10 = 34.62µs 58.7𝜆
ϴ= 𝐿
(degrees)
Assuming the horizontal resolution stay at
(58.7)(0.164)
428 lines, the new bandwidth is ϴ= 25

trace time x BW x 2 = 428 ϴ = 0.385 degrees

BW = 428 / (trace time x 2) = 6.2MHz


Where: Ro = true earth’s radius = 6370 km
𝑐 3𝑥108 𝑚/𝑠
𝜆= 𝑓
= 6𝑥109 𝐻𝑧
= 0.05m = 0.164ft Ns = surface refractivity = 300

∴ The beamwidth of a fully illuminated Solution:


microwave passive repeater is 0.385o.
Re = Ro [1-0.04665e0.005577Ns]-1
PROBLEM 3
Re = 6370[1-0.04665e0.005577(300)]-1
In microwave communications, what is the
minimum received carrier power in dBm if Re = 8477 km ≈ 8500km
the minimum carrier-to-noise requirement
for a receiver with a 10MHz noise bandwidth ∴ The effective earth’s radius is
is 24dB? approximately 8500km.

Notations: PROBLEM 5
Cmin = minimum received carrier power
In space diversity, what is the antenna
(dBm)
separation required for optimum operation if
𝐶
= carrier-to-noise ratio (dB) the operating frequency is 8 GHz and the
𝑁
path length is 40 miles?
N = thermal noise (dB)
Notations:
Solution:
𝐶 S = vertical separation distance between
Cmin = 𝑁(dB) + N(dBm)
two antennas (m)
Cmin = 24(dB) + (-104 dBm)
𝜆= signal wavelength (m)
Cmin = -80 dBm
Re = effective earth’s radius (km)
Where:
L = path length or distance (km)
N = -174 dBm +10log(10x106)

N = -174 dBm + 70dB = -104 dBm Ro = true earth’s radius = 6371 km

∴ The minimum received carrier power k = correction factor for relatively flat earth
is -80 dBm.
k = 4/3 (typical value)
PROBLEM 4
Solution:
What is the effective earth’s radius when Ns
= 300? 3𝜆𝑅𝑒
S= 𝐿
Notations:
3(37.5𝑥10−3 )(8495.52𝑥103 )
Re = effective earth’s radius (km) S= 64.36𝑥103
S = 14.85 m Notations:

Where: Let h = clearance from an obstacle (m)

𝜆=𝑓=
𝑐 3𝑥108 𝑚/𝑠
= 37.5 𝑚𝑚 R = radius or height of first Fresnel Zone
8𝑥109 𝐻𝑧
(m)
L = 40 miles = 64.36km
N = number of Fresnel Zones (FZ)
Re = kRo
N = 1 for first FZ = 2 for 2nd FZ, etc.
Re = (4/3) (6371)
Solution:
Re = 8495.52 km
h = 0.6(Rm)
∴ The antenna separation is 14.85 m.
h = 0.6(30.58)
PROBLEM 6
h = 18.35m
What is the correction factor for relatively flat
where:
earth if the surface refractivity is 250?
𝑛𝑑1(𝑘𝑚) 𝑑2(𝑘𝑚)
Notations: R(m) = 17.3 √ 𝑓𝐺𝐻𝑧 𝑑𝑘𝑚

k = correction factor for relatively flat earth 25(25)


R(m) = 17.3 √
4(50)
Ns = surface refractivity = 250
R(m) = 30.58m
Solution:
∴ The clearance from the obstacle is
k = (1-0.04665e0.005577Ns)-1
18.35m.
k = [1-0.04665e0.005577(250)]-1
PROBLEM 8
k = 1.23
In a certain microwave links, the following
parameters are given: Path distance = 40km,
∴ The correction factor is 1.23.
average terrain and climate operating
PROBLEM 7 frequency = 8GHz, reliability desired =
99.99%. Determine the fade margin to be
A microwave radio-link operated at assigned to this link.
frequency of 4GHz has a separation distance
of 50km between antennas. By how much Notations:
must the beam clear the obstacle?
d = path distance (km)

a = terrain characteristics
a = 1 for average characteristics PROBLEM 10

b = climate characteristics In a microwave communications systems, if


the minimum carrier-to-noise (C/N)
b = 0.25 for average climate requirement for a receiver with a 10MHz
bandwidth is 22dB, the minimum receive
f = frequency (GHz)
carrier power is ______.
R = system reliability
Notations:
Solution: 𝐶
𝑁
= carrier-to-noise ratio (dB)
FM = 30logdkm + 10log6abf – 10log(1-R)-70
Cmin = minimum receiver carrier power
FM = 30log40 +10log (6)(1) (0.25) (8)- (dBm)
10log (1=0.9999)-70
N = thermal noise power (dBm)
FM = 28.85 dB
Solution:
∴ The fade margin to be assigned to
𝐶
the link is 26.85 dB. 𝑁
(dB) = Cmin (dBm) – N(dBm)

PROBLEM 9 𝐶
Cmin(dBm) = 𝑁(dB) + NdBm

For a relatively flat earth, what is the


Cmin = 22 dB + (-103.98 dBm)
correction factor used when the surface
refractivity is 300? Cmin = -82 dBm

Notations: Where:

k = correction for relatively flat earth N = kTBW = (1.38x10-23) (290) (10x106)

Ns surface refractivity N = 4.002x10-14W

Solution: 4.002𝑥10−14
NdBm= 10log( 1𝑥10−3
) = −103.98
k = [1-0.04665e0.005577Ns]-1
PROBLEM 11
k = [1-0.04665e0.005577(300)]-1
In microwave communications systems, for a
k = 1.33 carrier frequency of 6GHz and a distance of
40km, determine the free-space path loss in
∴ The correction factor is 1.33. dB.
Notations: 3(
3𝑥103
)(8494.67𝑥103 )
10𝑥109
S= 30𝑥103
Lp = free space path loss (dB)
S = 25.48m
D = path distance (km)
Where:
f = frequency (GHz)
4
Re = 3(6731)
Solution:
Re = 8494.67 km
Lp = 92.4 +20logdkm + 20logfGHz
∴ The antenna separation is 25.48m.
Lp = 92.4 + 20log40 + 20log6
Question Bank 9: SATELLITE
Lp = 140dB
COMMUNICATIONS
∴ The free space path loss is 140dB.
PROBLEM 1
PROBLEM 12
What is the uplink receiver frequency if the
In space diversity, what is the antenna downlink transmitter is on channel 4 (3.84
separation required for optimum GHz)?
performance assuming a relatively flat earth
Notations:
with a path length of 30km at 10 GHz?
fu = uplink frequency (GHz)
Notations:
fd = downlink frequency (GHz)
Re = effective earth’s radius (km)
fo = transponder oscillator frequency (GHz)
k = correction factor for relatively flat earth
= 4/3 Solution:

Ro = true earth’s radius = 3960 miles = fu = fd + fo


6371 km
fu = 3.84 GHz + 2.225 GHz
L = path length or distance = 30km =
30x103m fu = 6.065 GHz

Solution: ∴ The uplink receiver frequency is 6.065


GHz.
3𝜆𝑅𝑒
S=
𝐿
PROBLEM 2
𝑐
3( )𝑅
S= 𝐿
𝑓
Calculate the required via net loss for a
satellite link with a one-way delay of 250 ms.
Notations: In satellite communications, what is the
height of a satellite from the earth’s surface
VNL = via net loss (dB) if the sidereal period is 20 hours?

T time for one-way delay (ms) Notations:

Solution: 𝑔 = 𝑎𝑐𝑐𝑒𝑙𝑒𝑟𝑎𝑡𝑖𝑜𝑛 𝑑𝑢𝑒 𝑡𝑜 𝑔𝑟𝑎𝑣𝑖𝑡𝑦 =


9.81 𝑥 10−3 𝑘𝑚/𝑠 2
VNL = (0.2t + 0.4) dB 𝑇 = 𝑠𝑖𝑑𝑒𝑟𝑒𝑎𝑙 𝑝𝑒𝑟𝑖𝑜𝑑 = 20 ℎ𝑜𝑢𝑟𝑠 = 72,000 𝑠
VNL = 0.2(250) + 0.4 𝑅 = 𝑒𝑎𝑟𝑡ℎ′ 𝑠 𝑟𝑎𝑑𝑖𝑢𝑠 = 6371 𝑘𝑚 =
3960 𝑚𝑖𝑙𝑒𝑠
VNL = 50.4 dB
Solution:
∴The required via net loss is 50.4 dB.
3 𝑔𝑇 2 𝑅2
ℎ=√ −𝑅
PROBLEM 3 4𝜋2

A satellite transmitter operates at 4 GHz with 3 (9.81 𝑥 10−3 )(72000)2 (6371)2


ℎ=√ 4𝜋2
− 6371
an antenna gain of 40 dBi. The receiver
40,000 km away has an antenna gain of 50 ℎ = 31,022.5 𝑘𝑚
dBi If the transmitter has a power of 8 W,
find (ignoring feedline losses and mismatch) ⸫ The height of the satellite is 31,022.5
the EIRP in dBW. km.

Notations: PROBLEM 5

𝐸𝐼𝑅𝑃 = Find the orbital period of a satellite in a


𝑒𝑓𝑓𝑒𝑐𝑡𝑖𝑣𝑒 𝑖𝑠𝑜𝑡𝑟𝑜𝑝𝑖𝑐 𝑟𝑎𝑑𝑖𝑎𝑡𝑒𝑑 𝑝𝑜𝑤𝑒𝑟 (𝑑𝐵𝑊) circular orbit 600 km above the surface of the
earth.
PR = radiated power to the antenna from the
transmitter (dBW) Notations:
𝑚
𝐺𝑇 = 𝑡𝑟𝑎𝑛𝑠𝑚𝑖𝑡 𝑎𝑛𝑡𝑒𝑛𝑛𝑎 𝑔𝑎𝑖𝑛 (𝑑𝐵) 𝑣 = 𝑠𝑎𝑡𝑒𝑙𝑙𝑖𝑡𝑒 𝑣𝑒𝑙𝑜𝑐𝑖𝑡𝑦 𝑖𝑛 𝑜𝑟𝑏𝑖𝑡 ( 𝑠 )

Solution: 𝑅 = 𝑒𝑎𝑟𝑡ℎ′ 𝑠 𝑎𝑣𝑒𝑟𝑎𝑔𝑒 𝑟𝑎𝑑𝑖𝑢𝑠 (𝑘𝑚) =


𝐸𝐼𝑅𝑃𝑑𝐵𝑊 = 𝑃𝑅(𝑑𝐵𝑊) − +𝐺𝑇(𝑑𝐵) 6371 𝑘𝑚

ℎ = 𝑠𝑎𝑡𝑒𝑙𝑙𝑖𝑡𝑒 ℎ𝑒𝑖𝑔ℎ𝑡 (𝑘𝑚) = 600 𝑘𝑚


With no losses,
8𝑊 𝑑 = 𝑑𝑖𝑠𝑡𝑎𝑛𝑐𝑒 =
𝑃𝑅(𝑑𝐵𝑊) = 𝑃 𝑇(𝑑𝐵𝑊) = 10 log ( ) =9
1𝑊 𝑐𝑖𝑟𝑐𝑢𝑚𝑓𝑒𝑟𝑒𝑛𝑐𝑒 𝑜𝑓 𝑡ℎ𝑒 𝑜𝑟𝑏𝑖𝑡 (𝑚)
𝐸𝐼𝑅𝑃(𝑑𝐵𝑊) = 9 + 40 = 49 𝑑𝐵𝑊 𝑇 = 𝑠𝑎𝑡𝑒𝑙𝑙𝑖𝑡𝑒 𝑝𝑒𝑟𝑖𝑜𝑑

⸫ The EIRP is 49 dBW. Solution:

PROBLEM 4 4𝑥1011
𝑣 = √ (𝑅+ℎ)
𝐶
4𝑥1011 = 100𝑑𝐵
𝑣 = √(6371+600) 𝑁

𝑚 where:
𝑣 = 7574.997 𝑠
𝐸𝐼𝑅𝑃(𝑑𝐵𝑊) = 𝑃 𝑇(𝑑𝐵𝑊) + 𝐺𝑇(𝑑𝐵)
𝑑 2𝜋(𝑅+ℎ)
𝑣= 𝑇
= 𝑇 50 𝑊
𝐸𝐼𝑅𝑃(𝑑𝐵𝑊) = 10 log ( )+ 40
1𝑊
2𝜋(6371+600)𝑥103 𝑚
𝑇=
7574.997𝑚/𝑠 𝐸𝐼𝑅𝑃(𝑑𝐵𝑊) = 57 𝑑𝐵𝑊
𝑇 = 5782.19 𝑠 𝑀 = 20 𝑑𝐵
𝑇 = 1.6 ℎ𝑜𝑢𝑟𝑠 𝐿𝑃 = 92.4 + 20 log 38000 + 20 log 12
⸫ The orbital period of a satellite is 1.6 𝐿𝑃 = 205.6 𝑑𝐵
hours.
⸫ The carrier-to-noise ratio at the
PROBLEM 6 receiver is 100 dB.
Past ECE Board Problem PROBLEM 7
A satellite earth receiving installation with a Past ECE Board Problem
figure of merit of 20 dB is used as a ground
terminal to receive a signal from a satellite at Calculate the free space loss between a
a distance of 38,000 km. The satellite has a satellite transmitter and a satellite receiver at
Tx power of 50 W and an antenna gain of 40 a distance of 25,000 km using 6 GHz.
dB. Assuming negligible losses between Notations:
satellite Tx and its antenna, calculate the C/N
at the Rx for a BW of 1 MHz using 12 GHz 𝐿𝑃 = 𝑓𝑟𝑒𝑒 𝑠𝑝𝑎𝑐𝑒 𝑝𝑎𝑡ℎ 𝑙𝑜𝑠𝑠 (𝑑𝐵)
frequency. (Boltzmann’s constant is equal to 𝑓 = 𝑓𝑟𝑒𝑞𝑢𝑒𝑛𝑐𝑦 (𝐺𝐻𝑧)
-228.6 dBW)
𝑑 = 𝑝𝑎𝑡ℎ 𝑑𝑖𝑠𝑡𝑎𝑛𝑐𝑒(𝑘𝑚)
Notations:
Solution:
𝐶
𝑁
= 𝑐𝑎𝑟𝑟𝑖𝑒𝑟 − 𝑡𝑜 − 𝑛𝑜𝑖𝑠𝑒 𝑟𝑎𝑡𝑖𝑜 (𝑑𝐵)
𝐿𝑃 = 92.4 + 20 𝑙𝑜𝑔𝑓𝐺𝐻𝑧 + 20 log 𝑑𝑘𝑚
𝐸𝐼𝑅𝑃 =
𝐿𝑃 = 92.4 + 20 log 6 + 20 log 25,000
𝑒𝑓𝑓𝑒𝑐𝑡𝑖𝑣𝑒 𝑖𝑠𝑜𝑡𝑟𝑜𝑝𝑖𝑐 𝑟𝑎𝑑𝑖𝑎𝑡𝑒𝑑 𝑝𝑜𝑤𝑒𝑟 (𝑑𝐵𝑊)
𝐿𝑃 = 195.92 𝑑𝐵
𝑀 = 𝑟𝑒𝑐𝑒𝑖𝑣𝑒𝑟 𝑓𝑖𝑔𝑢𝑟𝑒 𝑜𝑓 𝑚𝑒𝑟𝑖𝑡 (𝑑𝐵)
𝑑𝑘𝑚 = √(𝑅 + ℎ)2 − 𝑅 2 cos 2 𝛽 − 𝑅 sin 𝛽
𝐿𝑃 = 𝑓𝑟𝑒𝑒 𝑠𝑝𝑎𝑐𝑒 𝑝𝑎𝑡ℎ 𝑙𝑜𝑠𝑠 (𝑑𝐵)
𝑑𝑘𝑚 =
Solution:
√(6300 + 36000)2 − (6400)2 cos 2 45 −
𝐶
𝑁
= 𝐸𝐼𝑅𝑃(𝑑𝐵𝑊) + 𝑀 − 𝐿𝑃 + 228.6 6400𝑠𝑖𝑛45
𝐶
= 57 + 20 − 205.6 + 228.6 𝑑𝑘𝑚 = 38115.34𝑘𝑚
𝑁
⸫ The free space loss is 195.92 dB.
PROBLEM 8 𝑑𝑘𝑚 =
√(6300 + 36000)2 − (6400)2 cos 2 45 −
Past ECE Board Problem
6400𝑠𝑖𝑛45
An uplink Tx of a satellite system operates at
𝑑𝑘𝑚 = 38115.34𝑘𝑚
6 GHz with a Tx power of 12 kW and an
antenna gain of 50 dBi. The receiver on the ⸫ The received signal strength is 38.8
geostationary satellite has an antenna gain dBm.
of 40 dBi. The elevation angle to the satellite
PROBLEM 9
from the ground is 45 deg., calculate the
received signal strength at the satellite. Past ECE Board Problem
(Height of satellite above the equator is
36,000 km., earth radius is 6400 km) A satellite transmitter with 8 watts power
operates at 4 GHz with an antenna gain of 40
Notations: dBi. The receiver, 40,000 km away has an
𝑃𝑅 = 𝑟𝑒𝑐𝑒𝑖𝑣𝑒𝑑 𝑠𝑖𝑔𝑛𝑎𝑙 𝑠𝑡𝑟𝑒𝑛𝑔𝑡ℎ (𝑑𝐵𝑚) antenna gain of 50 dBi. If the overall loss is
9 dB, find the ERP.
𝑃𝑇 = 𝑡𝑟𝑎𝑛𝑠𝑚𝑖𝑡 𝑝𝑜𝑤𝑒𝑟 (𝑑𝐵𝑚)
Notations:
𝐿𝑃 = 𝑓𝑟𝑒𝑒 𝑠𝑝𝑎𝑐𝑒 𝑝𝑎𝑡ℎ 𝑙𝑜𝑠𝑠 (𝑑𝐵)
𝐸𝑅𝑃 = 𝑒𝑓𝑓𝑒𝑐𝑡𝑖𝑣𝑒 𝑟𝑎𝑑𝑖𝑎𝑡𝑒𝑑 𝑝𝑜𝑤𝑒𝑟 (𝑑𝐵𝑊)
𝑑 = 𝑠𝑎𝑡𝑒𝑙𝑙𝑖𝑡𝑒 𝑟𝑎𝑛𝑔𝑒(𝑘𝑚)
𝑃𝑇 = 𝑠𝑎𝑡𝑒𝑙𝑙𝑖𝑡𝑒 𝑡𝑟𝑎𝑛𝑠𝑚𝑖𝑡 𝑝𝑜𝑤𝑒𝑟 (𝑑𝐵𝑊)
Solution:
𝑆𝑜𝑙𝑢𝑡𝑖𝑜𝑛:
𝑃𝑅(𝑑𝐵𝑚) = 𝑃 𝑇(𝑑𝐵𝑚) + 𝐺𝑇(𝑑𝐵𝑖) − 𝐿𝑃(𝑑𝐵) +
𝐸𝑅𝑃𝑑𝐵𝑊 = 𝑃 𝑇(𝑑𝐵𝑊) − 𝐹𝑖𝑥𝑒𝑑 𝐿𝑜𝑜𝑠𝑑𝐵 + 𝐺𝑎𝑖𝑛𝑑𝐵
𝐺𝑅(𝑑𝐵𝑖)
𝐸𝑅𝑃𝑑𝐵𝑊 = 9 𝑑𝐵𝑊 − 9 𝑑𝐵 + 40 𝑑𝐵
𝑃𝑅(𝑑𝐵𝑚) = 70.79 𝑑𝐵𝑚 + 50 − 199.56 + 40
𝐸𝑅𝑃𝑑𝐵𝑊 = 40 𝑑𝐵𝑊
𝑃𝑅(𝑑𝐵𝑚) = −38,8 𝑑𝐵𝑚
where:
8𝑊
𝑃𝑇 = 10 log (1𝑊) = 9.03 𝑑𝐵𝑊
where:
12 𝑥 103 𝐹𝑖𝑥𝑒𝑑 𝑙𝑜𝑠𝑠(𝑡𝑟𝑎𝑛𝑠𝑚𝑖𝑡𝑡𝑒𝑟) = 9 𝑑𝐵
𝑃𝑇(𝑑𝐵𝑚) = 10 log (1 𝑥 10−3 )
⸫ The ERP is 40 dBW.
𝑃𝑇(𝑑𝐵𝑚) = 70.79 𝑑𝐵𝑚
PROBLEM 10
𝐿𝑃 = 92.4 + 20 𝑙𝑜𝑔𝑓𝐺𝐻𝑧 + 20 log 𝑑𝑘𝑚
Past ECE Board Problem
𝐿𝑃 = 92.4 + 20 log 6 + 20 log 38115.34
In a satellite communications system, for a
𝐿𝑃 = 199.58 𝑑𝐵 total transmit power of 5000 watts,
𝑑𝑘𝑚 = √(𝑅 + ℎ)2 − 𝑅 2 cos 2 𝛽 − 𝑅 sin 𝛽 determine the energy per bit for a
transmission rate of 50 Mbps expressed in
dBW.
Notations: Question Bank 10: MULTIPLEXING

Eb = energy per bit (J/bit or J or dBW when PROBLEM 1


referred to 1 W)
The basic European TDM telephone signal
𝑃𝑇 = 𝑡𝑜𝑡𝑎𝑙 𝑡𝑟𝑎𝑛𝑠𝑚𝑖𝑡 𝑝𝑜𝑤𝑒𝑟 (𝑊) combines 30 voice channels with 2 signaling
channels that have the same date rate as the
𝑓𝑏 = 𝑏𝑖𝑡 𝑟𝑎𝑡𝑒 (𝑏𝑝𝑠)
voice channels. The sampling rate is 8 kHz
Solution: and there are 8 bits per sample for each
𝑃𝑇 5000 voice channel. Calculate the total bit rate for
𝐸𝑏 = 𝑓𝑏
= 50𝑥106 this signal.

𝐸𝑏 = 100𝑢𝐽/𝑏𝑖𝑡 Notations:
In dBW, 𝑓𝑏 = 𝑏𝑖𝑡 𝑟𝑎𝑡𝑒 (𝑏𝑝𝑠)
100𝑥10−6 𝐶 = 𝑡𝑜𝑡𝑎𝑙 𝑛𝑢𝑚𝑏𝑒𝑟 𝑜𝑓 𝑐ℎ𝑎𝑛𝑛𝑒𝑙𝑠 = (30 + 2)
𝐸𝑏 = 10 log( 1𝑊
)
𝑏 = 𝑛𝑢𝑚𝑏𝑒𝑟 𝑜𝑓 𝑏𝑖𝑡𝑠 𝑝𝑒𝑟 𝑠𝑎𝑚𝑝𝑙𝑒 = 8
𝐸𝑏 = −40 𝑑𝐵𝑊
𝑓𝑠 = 𝑠𝑎𝑚𝑝𝑙𝑖𝑛𝑔 𝑟𝑎𝑡𝑒 (𝐻𝑧) = 8 𝑘𝐻𝑧
⸫ The energy per bit is -40 dBW.
Solution:
PROBLEM 11
𝑓𝑏 = 𝐶𝑏𝑓𝑠 = [(32)(8)](8000)
At what height is the satellite from the earth’s
surface if its sidereal period is 18 hours? 𝑓𝑏 = 2.048 𝑀𝑏𝑝𝑠

Notations: ⸫ The total bit rate is 2.048 Mbps.

ℎ = 𝑠𝑎𝑡𝑒𝑙𝑙𝑖𝑡𝑒 ℎ𝑒𝑖𝑔ℎ𝑡 (𝑘𝑚) PROBLEM 2

𝑇 = 𝑠𝑎𝑡𝑒𝑙𝑙𝑖𝑡𝑒 𝑝𝑒𝑟𝑖𝑜𝑑 = 18 ℎ𝑜𝑢𝑟𝑠 = 64800 𝑠 Find the frequency to which a 400 Hz


baseband frequency will be translated, for
𝑅 = 𝑒𝑎𝑟𝑡ℎ′ 𝑠 𝑎𝑣𝑒𝑟𝑎𝑔𝑒 𝑟𝑎𝑑𝑖𝑢𝑠 = 6371 𝑘𝑚
channel of 8 of a group.
𝑔 = 𝑎𝑐𝑐𝑒𝑙𝑒𝑟𝑎𝑡𝑖𝑜𝑛 𝑑𝑢𝑒 𝑡𝑜 𝑔𝑟𝑎𝑣𝑖𝑡𝑦 =
Notations:
9.81 𝑥 10−3 𝑘𝑚/𝑠 2
𝑓𝑐 = 𝑐𝑎𝑟𝑟𝑖𝑒𝑟 𝑓𝑟𝑒𝑞𝑢𝑒𝑛𝑐𝑦 (𝐻𝑧)
Solution:
𝑓𝑖 = 𝑖𝑑𝑒𝑎𝑙 (𝑜𝑟𝑖𝑔𝑖𝑛𝑎𝑙 𝑠𝑝𝑒𝑐𝑡𝑟𝑢𝑚) (𝐻𝑧)
3𝑔𝑇 2 𝑅2
ℎ= √ 2 −𝑅
4𝜋 𝑓𝑜𝑢𝑡 = 𝑜𝑢𝑡𝑝𝑢𝑡 𝑓𝑟𝑒𝑞𝑢𝑒𝑛𝑐𝑦 (𝐻𝑧)
3 (9.81 𝑥 10−3 )(64800)2 (6371)2 𝑛 = 𝑐ℎ𝑎𝑛𝑛𝑒𝑙 𝑛𝑢𝑚𝑏𝑒𝑟
ℎ=√ 4𝜋2
− 6371
Solution:
ℎ = 28,486 𝑘𝑚
For channel 8,
⸫ The height of the satellite is 28,486
km. 𝑓𝑐 = 112 − 4(𝑛) = 112 − 4(8)

𝑓𝑐 = 80 𝑘𝐻𝑧
𝑓𝑜𝑢𝑡 = 𝑓𝑐 − 400 𝐻𝑧 ⸫ The total noise load ratio is 12.78 dB.

𝑓𝑜𝑢𝑡 = 80 𝐻𝑧 − 400 𝐻𝑧 PROBLEM 5


𝑓𝑜𝑢𝑡 = 79.6 𝑘𝐻𝑧 Determine the carrier frequency of channel
five in frequency division multiplexing.
⸫ The output frequency to which the
baseband frequency will be translated Notations:
is 79.9 kHz.
𝑓𝑐 = 𝑐𝑎𝑟𝑟𝑖𝑒𝑟 𝑓𝑟𝑒𝑞𝑢𝑒𝑛𝑐𝑦 (𝐻𝑧)
PROBLEM 3
𝑛 = 𝑐ℎ𝑎𝑛𝑛𝑒𝑙 𝑛𝑢𝑚𝑏𝑒𝑟
For an FDM system with 75% speech loading
Solution:
and 25% data loading, determine the rms
power level required as recommended by 𝑓𝑐 = (112 − 4𝑛) 𝑘𝐻𝑧
most manufacturers for 600 voice band
𝑓𝑐 = [112 − 4(5)]
channels.
𝑓𝑐 = 92 𝑘𝐻𝑧
Notations:
⸫ The carrier frequency is 92 kHz.
𝑃𝑟𝑚𝑠 = 𝑟𝑚𝑠 𝑝𝑜𝑤𝑒𝑟 𝑙𝑒𝑣𝑒𝑙 (𝑑𝐵)
PROBLEM 6
𝑁 = 𝑛𝑢𝑚𝑏𝑒𝑟 𝑜𝑓 𝑣𝑖𝑜𝑐𝑒 𝑏𝑎𝑛𝑑 𝑐ℎ𝑎𝑛𝑛𝑒𝑙
Determine the carrier frequency of group five
Solution:
in frequency division multiplexing.
𝑃𝑟𝑚𝑠 = −11 + 10 log 𝑁
Notations:
𝑃𝑟𝑚𝑠 = −11 + 10𝑙𝑜𝑔600
𝑓𝑐 = 𝑐𝑎𝑟𝑟𝑖𝑒𝑟 𝑓𝑟𝑒𝑞𝑢𝑒𝑛𝑐𝑦 (𝐻𝑧)
𝑃𝑟𝑚𝑠 = 16.78 𝑑𝐵
𝑛 = 𝑐ℎ𝑎𝑛𝑛𝑒𝑙 𝑛𝑢𝑚𝑏𝑒𝑟
⸫ The rms power level is 16.78 dB.
Solution:
PROBLEM 4
𝑓𝑐 = (372 − 48𝑛) 𝑘𝐻𝑧
Using CCIR recommendation, what is the
𝑓𝑐 = [372 − 48(5)]
noise load ratio for an FDM configuration
with 600 voice band channels? 𝑓𝑐 = 612 𝑘𝐻𝑧

Notations: ⸫ The carrier frequency is 612 kHz.

𝑁𝐿𝑅 = 𝑛𝑜𝑖𝑠𝑒 𝑙𝑜𝑎𝑑 𝑟𝑎𝑡𝑖𝑜 (𝑑𝐵) PROBLEM 7


𝑁 = 𝑛𝑢𝑚𝑏𝑒𝑟 𝑜𝑓 𝑣𝑖𝑜𝑐𝑒 𝑏𝑎𝑛𝑑 𝑐ℎ𝑎𝑛𝑛𝑒𝑙𝑠 Find the frequency to which a 400 Hz
baseband frequency will be translated for
Solution:
channel 7 of a group of FDM system.
𝑁𝐿𝑅 (𝑑𝐵) = −15 + 10 log 𝑁 Notations:
𝑁𝐿𝑅 (𝑑𝐵) = −15 + 10𝑙𝑜𝑔600 fc = carrier frequency (Hz)
𝑁𝐿𝑅(𝑑𝐵) = 12.78 𝑑𝐵 fi = ideal (original spectrum) (Hz)
fout = output frequency (Hz) ∴ The center frequency of its first
channel is 401.00625 MHz.
n = channel number
PROBLEM 9
Solution:
Let us consider a frequency range of
For channel 7,
405.0125 MHz to 405.0875 MHz and a 25 kHz
𝑓𝑐 = (112 − 4𝑛) 𝑘𝐻𝑧 channeling plan, determine the center
frequency of the second channel from the
𝑓𝑐 = 112 − 4(7) = 84 𝑘𝐻𝑧
lower limit.
𝑓𝑜𝑢𝑡 = 𝑓𝑐 − 𝑓𝑖 = 84 𝑘𝐻𝑧 − 400 𝐻𝑧 = 83.6 𝑘𝐻𝑧
Notations:

∴ The output frequency to which the fc2 = center frequency of its second channel
baseband frequency will be translated (Hz)
is 83.6 kHz. BW = bandwidth (Hz)

PROBLEM 8 fL = lower frequency limit (Hz)

If frequency range of 401.000 MHz to Solution:


401.050 MHz has to be channelized at 12.5
kHz between channels, what is the center of
its first channel from the lower limit?

Notations:
fc1 = center frequency of its first channel
(Hz)

BW = bandwidth (Hz)
𝐵𝑊
𝑓𝑐2 = 𝑓𝐿 + 𝐵𝑊 +
fL = lower frequency limit (Hz) 2

Solution: 25 𝑘𝐻𝑧
𝑓𝑐2 = 405.0125 𝑀𝐻𝑧 + 25 𝑘𝐻𝑧 +
2
= 405.05 𝑀𝐻𝑧

∴ The center frequency of its second


channel is 405.05 MHz.

PROBLEM 10

Assume a frequency range of 401.000 MHz


to 401.050 MHz, if you apply 25 kHz
channeling plan, determine the center
𝐵𝑊 frequency of the first channel from the lower
𝑓𝑐1 = 𝑓𝐿 +
2 limit.
12.5 𝑘𝐻𝑧
𝑓𝑐1 = 401.000 𝑀𝐻𝑧 +
2
= 401.00625 𝑀𝐻𝑧
Notations:
fc1 = center frequency of its first channel
(Hz)

BW = bandwidth (Hz)

fL = lower frequency limit (Hz)

Solution:
𝐵𝑊
𝑓𝑐2 = 𝑓𝐿 + 𝐵𝑊 +
2
25 𝑘𝐻𝑧
𝑓𝑐2 = 405.0125 𝑀𝐻𝑧 + 25 𝑘𝐻𝑧 +
2
= 405.05 𝑀𝐻𝑧

∴ The center frequency of its second


channel is 405.05 MHz.

PROBLEM 12
𝐵𝑊 Assuming a UHF frequency range of
𝑓𝑐1 = 𝑓𝐿 +
2 405.0125 MHz to 405.0875 MHz at 25 kHz
25 𝑘𝐻𝑧 channeling plan, how many channels can you
𝑓𝑐1 = 401.000 𝑀𝐻𝑧 + produce?
2
= 401.0125 𝑀𝐻𝑧
Notations:
∴ The center frequency of its first
N = total number of channels
channel is 401.0125 MHz.
fU = upper frequency limit (Hz)
PROBLEM 11
fL = lower frequency limit (Hz)
Assume a frequency range of 405.0125 MHz
to 405.0875 MHz, if you apply 25 kHz Solution:
channeling plan, determine the center
𝑓𝑈 − 𝑓𝐿
frequency of the second channel from the 𝑁=
𝐵𝑊
lower limit.
(405.0875 − 405.0125)𝑀𝐻𝑧
Notations: 𝑁=
25 𝑘𝐻𝑧
=3

fc2 = center frequency of its second channel ∴ The total number of channels is 3.
(Hz)
PROBLEM 13
BW = bandwidth (Hz)
Following CCITT’s recommendation (now
fL = lower frequency limit (Hz) ITU-T) for loading multichannel FDM
Solution: systems, what is the average power level
required when the number of voice band
channels is 240.
Notations: 𝑓𝑡 1 𝑦𝑎𝑟𝑑
𝑑 = 660 𝑚 (3.28 )( )
𝑚 3 𝑓𝑡
Pave = average power level (dB) = 721.6 𝑦𝑎𝑟𝑑𝑠
N = number of voice band channels
Where:
Solution: 𝑚
𝑣 = 3 × 108
For N ≥ 240 voice band channels 𝑠
4.4
𝑃𝑎𝑣𝑒 = −15 + 10𝑙𝑜𝑔𝑁 𝑡= 2
= 2.2 𝜇𝑠 (time to reach the object)

𝑃𝑎𝑣𝑒 = −15 + 10𝑙𝑜𝑔240 = 8.8 𝑑𝐵 ∴ The distance is 721.6 yards.

∴ The average power level is 8.8 dB. PROBLEM 3

Question Bank 9: RADAR What is the peak power of a radar pulse if


the pulse width is 0.1 µs, PRR is 900 and the
PROBLEM 1 average plate power input is 45 watts?
Given the pulse width of 8 microseconds and Notations:
a duty cycle of 8%, determine the pulse
repetition time of radar system. PRR = pulse repetition rate

Notations: PW = pulse width (µs)

PRT = pulse repetition time (µs) PRT = pulse repetition time (µs)

PW = pulse width (µs) Solution:

Solution:
𝑃𝑊 8 𝜇𝑠 𝐴𝑣𝑒𝑟𝑎𝑔𝑒 𝑃𝑜𝑤𝑒𝑟
𝑃𝑅𝑇 = = = 100 𝜇𝑠 𝐷𝑢𝑡𝑦 𝐶𝑦𝑐𝑙𝑒 =
𝐷𝑢𝑡𝑦 𝐶𝑦𝑐𝑙𝑒 0.08 𝑃𝑒𝑎𝑘 𝑃𝑜𝑤𝑒𝑟

∴ The pulse repetition time is 100 µs. 𝐴𝑣𝑒𝑟𝑎𝑔𝑒 𝑃𝑜𝑤𝑒𝑟 45 𝑊


𝑃𝑒𝑎𝑘 𝑃𝑜𝑤𝑒𝑟 = =
𝐷𝑢𝑡𝑦 𝐶𝑦𝑐𝑙𝑒 9 × 10−4
PROBLEM 2 = 50 𝑘𝑊
Find the distance in yards to an object if the Where:
display of a radar signal measures 4.4
𝑃𝑊
microseconds. 𝐷𝑢𝑡𝑦 𝐶𝑦𝑐𝑙𝑒 =
𝑃𝑅𝑇
Notations:
1.0 × 10−6
𝐷𝑢𝑡𝑦 𝐶𝑦𝑐𝑙𝑒 = = 9 × 10−4
d = distance (yards) 1.11 × 10−3

v = c = signal velocity = 3 x 108 m/s 1 1


𝑃𝑅𝑇 = = = 1.11 × 10−3 𝑠
𝑃𝑅𝑅 900
t = time (s)
∴ The peak power of the radar is 50
Solution: kW.
𝑑 = 𝑣𝑡 = (3 × 108 )(2.2 × 10−6 )
PROBLEM 4 PROBLEM 6

What is the Doppler shift in frequency A deep space radar has a noise figure of 20
caused by a vehicle moving towards a radar dB and IF bandwidth of 2.0 MHz. What is the
at 96 kph if the radar is operated at 10 minimum signal received for satisfactory
GHz? reception?

Notations: Notations:
fD = Doppler shift in frequency (Hz) Pmin = minimum power received (W)

Vr = velocity of the car (m/s) To = room temperature (°K)

fi = operating frequency (Hz) BW = Bandwidth (Hz)

c = wave velocity (m/s) F = noise factor

Solution: K = Boltzmann’s constant = 1.38 x 10-23


J/°K
2𝑉𝑟 𝑓𝑖 2(26.7)(10 × 109 )
𝑓𝐷 = = = 1.78 𝑘𝐻𝑧
𝑐 3 × 108 Solution:
Where: 𝑃𝑚𝑖𝑛 = 𝑘𝑇0 𝐵𝑊(𝐹 − 1)
𝑉𝑟 = 96 𝑘𝑝ℎ 𝑃𝑚𝑖𝑛 = (1.38 × 10−23 )(290)(2.0 × 106 )(100
− 1)
𝑘𝑚 1ℎ𝑟 1000𝑚 𝑚
𝑉𝑟 = 96 × × = 26.7
ℎ𝑟 3600 1𝑘𝑚 𝑠 𝑃𝑚𝑖𝑛 = 7.923 × 10−33 𝑊
∴ The Doppler shift in frequency is 1.78 Where:
kHz.
20
PROBLEM 5 𝐹 = 𝑙𝑜𝑔−1 ( ) = 100
10
A deep space radar of Air Transportation ∴ The minimum signal received is 7.923
Office has a duty cycle of 25%. What is the x 10-13 W.
PW if the PRT is 6 ms?
PROBLEM 7
Notations:
A space radar is used to detect unidentified
PW = pulse width (ms) flying object 500 km from the earth’s surface.
What is the peak pulse power of the radar if
PRT = pulse repetition time (ms)
the minimum receivable power is 10-9, the
Solution: capture area of the antenna is 10 m2, and the
target’s cross-sectional area is 30 m2 at 10
𝑃𝑊
𝐷𝑢𝑡𝑦 𝐶𝑦𝑐𝑙𝑒 = GHz operating frequency?
𝑃𝑅𝑇
𝑃𝑊 = (𝐷𝑢𝑡𝑦 𝐶𝑦𝑐𝑙𝑒)(𝑃𝑅𝑇)
Notations:

𝑃𝑊 = (0.25)(6 𝑚𝑠) = 1.5 𝑚𝑠 rmax = maximum radar range (m)

∴ The pulse width is 1.5 ms. Pt = peak pulse power of the radar (W)
Ao = capture area of the receiving antenna ∴ The radial velocity of the target is
(m2) 16.67 m/s.

S =radar cross section or effective area PROBLEM 9


(m2)
A deep space radar has a noise figure of 10
Pmin = minimum receivable power of the dB, IF bandwidth of 2.0 MHz, a +7 dBm
receiver (W) third order intercept and 0 dB S/N. What is
Solution: the minimum signal received for satisfactory
reception?
𝑃 𝐴 2𝑆
𝑟𝑚𝑎𝑥 = (4𝜋𝜆𝑡 2𝑜𝑃 )1/4
min Notations:
𝑟𝑚𝑎𝑥 4 (4𝜋𝜆2 𝑃min ) S = minimum signal received for
𝑃𝑡 =
𝐴𝑜 2 𝑆 satisfactory reception (dBm)
(500000)4 (4𝜋)(0.03)2 (1 × 10−9 ) NF = noise figure (dB)
𝑃𝑡 = [ ]
(3𝐶)2 (15)
= 52.4 𝑀𝑊 f = IF bandwidth (MHz)
S
∴ The peak pulse power of the radar is N
= signal-to-noise ratio (dB)
52.4 MW.
Solution:
PROBLEM 8
S
S = -174 dBm + NF +10 log f + N
Calculate the radial velocity of the target, if
the Doppler frequency seen by a stationary S = -174 + 10 +10log(2x106 ) + 0
radar is 1 kHz and the radar transmit a CW
at a frequency of 9 GHz. S = -101 dBm

Notations: The minimum signal received for


satisfactory reception is -101 dBm.
Vr = radial velocity (m/s)

fd = Doppler frequency (Hz) PROBLEM 10

f = operating frequency of the radar (Hz) A deep space radar has a noise figure of 10
dB, IF bandwidth of 2.0 MHz, a +7 dBm third
𝜆 = signal wavelength (m) order intercept and 0 dB S/N, and the
Solution: sensitivity level is -100 dBm. Calculate the
dynamic range of the receiver.
𝑐 3 × 108
𝜆= = = 0.033 𝑚 Notations:
𝑓 9 × 109
2𝑉𝑟 DR = dynamic range (dB)
𝑓𝑑 =
𝜆
Solution:
𝑓𝑑 𝜆 (1 × 103 )(0.033) 𝑚
𝑉𝑟 = = = 16.67 2
2 2 𝑠 DR  3
(input intercept – noise floor)

2
DR = 3[7-(-100)]
DR = 71.33 dB Question Bank 12: WAVEGUIDES

The dynamic range of the receiver is PROBLEM 1


71.33 dB.
What is the cut-off frequency of a regular
PROBLEM 11 waveguide with a width of 0.65 inch and a
height of 0.38 inch?
NASA’s deep space radar has the following
parameters: Notations:

Operating frequency --- 3 GHz fc = cut-off frequency (MHz)

Peak pulse power ------- 20 MW a = width (m)

Antenna diameter ------- 32 m Solution:

Target cross section ----- 1.5 m2 300


fc =
2a
Noise factor --------------- 1.5
300
fc =
Receiver bandwidth ------ 8 kHz 2[(0.65)(2.54x10−2 )]
What is the maximum range of the radar? fc = 9,085 MHz

Notations: The cut-off frequency of a regular


waveguide is 9,085 MHz.
Pt = peak value of transmitted pulse power
(W) PROBLEM 2

D = antenna diameter (m) Determine the cut-off wavelength of a


circular waveguide having an internal radius
S = radar cross section or effective area
of 10cm if the solution of a Bessel function
(m2 )
equation (kr) is 3.83.
BW = Bandwidth (Hz)
Notations:
 = wavelength (m)
o = cut-off wavelength (m)
NF = Noise Factor
r = circular waveguide internal radius (m)
Solution:
kr = solution of a Bessel function equation
Pt D4 S
rmax = 48 [ ]1/4 Solution:
BW2 {NF−1)

20x106 (324 )(1.5) 1/4 o =


2πf
rmax = 48 [ ] kr
8x103 (0.1)2 (1.5 − 1)
2π(10x10−2 )
rmax = 45, 202 km o =
3.83
o = 164.05 mm
 The cut-off wavelength of a circular Solution:
waveguide is 164.05 mm.
The typical operating frequency is given by
PROBLEM 3 fc 9.085
f= 0.7
= 0.7
= 12.98 GHz
The cut-off frequency of a TE10 mode is 3.75
GHz in an air-dielectric waveguide with an where:
inside cross-section of 2 cm by 4 cm. What is 300 300
fc = = = 9.085 GHz
the characteristic impedance of this 2a 2[0.65 in x
0.0254 𝑚
𝑖𝑛
]
waveguide at the operating frequency of 10
GHz? The typical operating frequency is
12.98 GHz.
Notations:
PROBLEM 5
Zo = characteristic Impedance (Ω)
A rectangular waveguide used for microwave
fc = cut-off frequency (GHz) transmission has a width of 1.4 inches and a
height of 0.8 inches. All signals above _____
f = operating frequency (GHz)
GHz will be passed by the waveguide.
Solution:
Notations:
377
Z0 = o = cut-off wavelength (m)
√1 − (fc )2
f fc = cut-off frequency (GHz)
377
Z0 = a = width (m)
√1 − (3.75)2
10 Vp = propagation velocity (m/s)

Z0 = 406.68 Ω Solution:
The characteristic impedance of this Assuming dominant mode,
waveguide is 406.68 Ω. 2a
o = m
; m=1
PROBLEM 4
2(1.4)
o = = 2.8 in = 0.071 m
A rectangular waveguide has a width of 0.65 1
in and a height of 0.38 in. what is the typical Vp 3x108
fc = = = 4.22 GHz
operating frequency of this waveguide? o 0.071

Notations: The cut-off frequency is 4.22 GHz.

f = operating frequency (GHz) PROBLEM 6

a = width (m) A rectangular waveguide has a width of 1.2


in and height of 0.7 in. the waveguide will
fc = cut-off frequency (GHz)
pass all signals above _______ GHz.
Notations: PROBLEM 8

o = cut-off wavelength (m) A cassegrain feed sectoral horn antenna uses


a waveguide with an inner cross section of
fc = cut-off frequency (GHz)
10 by 20 mm. Calculate the cut-off frequency
Vp = propagation velocity (m/s) for TE10 mode.

Solution: Notations:

Assuming dominant mode, fc = cut-off frequency (GHz)

o =
2a
; m=1 a = waveguide width (m)
m

2(1.2) c = wave velocity = 3x108 m/s


o = 1
= 2.4 in = 0.061 m
Solution:
Vp 3x108
fc = o
= = 4.9 GHz 𝑐 3x108
0.061
fc = 2a
= 2(0.020)
= 7.5 GHz
The cut-off frequency is 4.9 GHz.
The cut-off frequency for 𝐓𝐄𝟏𝟎 mode
PROBLEM 7 is 7.5 GHz.
A rectangular waveguide has a width of 0.6 PROBLEM 9
inch. Calculate the waveguide but-off
frequency. A slot antenna is used in a waveguide to
radiate EM wave. Suppose the operating
Notations: frequency of the signal is 12 GHz and the cut-
off frequency of the waveguide is 7.5 GHz,
o = cut-off wavelength (m)
how long should the antenna slot be installed
fc = cut-off frequency (GHz) in a waveguide?

Vp = propagation velocity (m/s) Notations:

Solution: L = length of the antenna slot (mm)

Assuming dominant mode, 𝑔 = guide wavelength (m)


2a
o = ; m=1  = signal wavelength (m)
m

2(0.6) fc = cut-off frequency (GHz)


o = 1
= 1.2 in = 0.03048 m
f = signal frequency (GHz)
Vp 3x108
fc = o
= = 9.84 GHz
0.03048 c = wave velocity (m/s)
The waveguide cut-off frequency is Solution:
9.84 GHz.
𝑔 0.032
L= 2
= 2
= 0.016 m
 c operating frequency is 6 GHz. Determine
𝑔 = =
f f
√1−( c )2
f
𝑓√1−( c )2
f the guide wavelength for the dominant
mode.

Notations:
3x108
𝑔 = = 0.032 m
12𝑥109 √1−(
7.5 2
12
) 𝑔 = guide wavelength (m)

The length of the antenna slot is 16 𝑐 = cutoff wavelength (m)


mm.  = signal wavelength (m)
PROBLEM 10
f = signal frequency (GHz)
A wave is propagated in a rectangular c = wave velocity (m/s)
waveguide with plane separation of 6 cm by
3 cm (inside measurements). Calculate the Solution:
cut-off frequency of the TE11 mode.
For dominant mode, m = 1
Notations: 2a 2(5)
𝑐 = m
= 1
= 10 cm
Tm,n = waveguide mode
𝑐 3x108
=𝑓= = 5 cm
m = 1 and n = 1 6x109

 5
fc = cut-off frequency (GHz) 𝑔 = 
= 5
= 15. 8 cm
√1−( )2 𝑓√1−( )2
𝑐 10
a = widest separation between walls (m)
The guide wavelength for the
b = separation between walls (m) dominant mode is 15.8 cm.
Solution: PROBLEM 12
Rectangular waveguide TE11 A rectangular waveguide with inside
measurement of 5 cm x 2 cm and the
c m 2 n 2
fc = √( ) + ( ) operating frequency is 6 GHz. Calculate the
2 a b corresponding group velocity.

3x108 1 2 1 2 Notations:
fc = √( ) + ( )
2 0.06 0.03 𝑔 = guide wavelength (m)

fc = 5.6 GHZ 𝑐 = cutoff wavelength (m)


The cut-off frequency of the 𝐓𝐄𝟏𝟏  = signal wavelength (m)
mode is 5.6 GHz.
f = signal frequency (GHz)
PROBLEM 11
c = wave velocity (m/s)
A rectangular waveguide with inside
measurement of 5 cm x 2 cm and the
Solution: 2𝜋𝑟
𝜆𝑜 = (𝑘𝑟) =
2𝜋(2.5)
= 8.54
(1.84)
𝑐
 = 𝑓 = 5 cm
𝑘𝑟 = 1.84 for TE11 mode
𝑐 = 2𝑎 = 2(5) = 10 𝑐𝑚 ∴ The characteristic impedance of the
  guide in a dominant mode is 394 Ω.

= 5
= 0.866
√1−( )2 √1−( )2
𝑐 10 PROBLEM 14

 2 A rectangular waveguide has an inside


𝑉𝑔 = 𝑐 (√1 − ( ) = 3x108 (0.866)
o measurement of 5.0 cm by 2.5 cm. Calculate
𝑉𝑔 = 2.6x108 m/s the cutoff wavelength, in a standard
rectangular waveguide, for the TM11 mode.
The group velocity is 𝟐. 𝟔𝐱𝟏𝟎𝟖 𝐦/𝐬.
Notations:
PROBLEM 13
Tmn = wave guide mode
A circular waveguide is used for conical horn
m = 1 and n = 1
antenna. If the internal diameter of the
waveguide is 5cm for a 12 GHz operating λo = cutoff wavelength (cm)
frequency, what is the characteristic
a = widest separation between walls (cm)
impedance of the guide in a dominant mode?
b = separation between walls (cm)
Notations:
Solution:
Zo = characteristic impedance of the guide
(Ω) 𝜆𝑜 =
2
𝑚 𝑛
√( 𝑎 )2 +(𝑏 )2
α = characteristic impedance of free space
(Ω) 𝜆𝑜 =
2
1 1
√( )2 +( )2
λo = cutoff wavelength (m) 5 2

λ = signal wavelength (m) 𝜆𝑜 = 3.71 𝑐𝑚

r = circular waveguide internal radius (m) ∴ The cutoff wavelength is 3.71 cm.

kr = solution of a Bessel function equation PROBLEM 15

Solution: A rectangular waveguide with a wall


separation of 2.5 cm, propagate a signal with
𝛼
𝑍𝑜 = unknown frequency. If the characteristic
2
√1−( 𝜆 ) impedance of the waveguide is 600 Ω, what
𝜆𝑜
is the frequency of the signal in the dominant
120𝜋
𝑍𝑜 = 2
mode?
√1−( 2.5 )
8.54
Notations:
𝑍𝑜 = 394 Ω
Zo = characteristic impedance of the guide
(Ω)
α = characteristic impedance of free space Solution:
(Ω) 2𝑎 2(2.5)
𝜆𝑜 = = = 5 𝑐𝑚
𝑚 1
λo = cutoff wavelength (m)
3×108
λ = signal wavelength (m) 𝜆 = 2.5×109 = 12 𝑐𝑚

𝑙
a = widest separation between walls (cm) 𝛼𝑣 = 54.5 (𝜆 ) 𝑑𝐵
𝑜

f = signal frequency (Hz) 1.5 𝑐𝑚


𝛼𝑣 = 54.5 ( 5 𝑐𝑚 ) 𝑑𝐵
Solution:
𝛼𝑣 = 16.35 𝑑𝐵
For dominant mode m = 1
∴ The voltage attenuation is 16.35 dB.
2𝑎 2(2.5)
𝜆𝑜 = = = 5 𝑐𝑚
𝑚 1 PROBLEM 17
𝛼 2 𝜆 2
(𝑍 ) =1− (𝜆 ) Suppose the two waveguide circular and
𝑜 𝑜
rectangular is to have the same cutoff
𝜆 2 120𝜋 wavelength for its dominant mode. What is
(𝜆 ) = 1 − ( 600 ) = 0.605
𝑜
the width of the rectangular waveguide if the
𝜆
= 0.78 radius of the circular waveguide is 3 cm?
𝜆𝑜
Notation:
𝜆 = 𝜆𝑜 (0.78) = (0.78)(5) = 3.89 𝑐𝑚
𝑐 3×108
λc(cir) = cutoff wavelength of circular guide
𝑓 = 𝜆 = 0.0389 = 7.7 𝐺𝐻𝑧 (m)

∴ The frequency of the signal in the λc(rec) = cutoff wavelength of rectangular


dominant mode is 7.7 GHz. guide (m)

PROBLEM 16 λ = signal wavelength (m)

A wave is propagated in the dominant mode a = width (cm)


using rectangular waveguide having a = 2.5
r = circular waveguide internal radius (m)
cm and b = 1.25 cm and 2.5 GHz operating
frequency. What is the voltage attenuation kr = solution of a Bessel function equation
provided by a 150 mm length of waveguide?
Solution:
Notation:
For the dominant TE11 mode in a circular
λo = cutoff wavelength (cm) waveguide,

a = width (cm) 2𝜋𝑓 2𝜋𝑓


𝜆𝑐(𝑐𝑖𝑟) = (𝑘𝑟) = 1.84 = 3.41 𝑟
αv = voltage attenuation (dB)
Area of a circle: 𝐴 = 𝜋𝑟 2
l = waveguide length (cm)
In the rectangular waveguide, for TE10 mode
𝜆𝑐 =
2𝑎
=
2𝑎
= 2𝑎 n = number of terminals
𝑚 1

𝜆𝑐(𝑟𝑒𝑐) = 𝜆𝑐(𝑐𝑖𝑟) Solution:


𝑛(𝑛−1)
2𝜋𝑟 𝑁=
2𝑎 = 2
(𝑘𝑟)
20(20−1)
𝜋𝑟 𝜋(3) 𝑁= = 190
𝑎= (𝑘𝑟)
= 1.84
2

∴ The number of wires or links needed


𝑎 = 5.12 𝑐𝑚
is 190.
∴ The width of the rectangular
PROBLEM 3
waveguide is 5.12 cm.
On a 10 Mbps Ethernet packet format, how
Question Bank 13: DIGITAL fast can a 1500-byte block of data be
COMMUNICATIONS transmitted?
Solution:
PROBLEM 1
1
In data communications, how many errors Time for 1 bit = 1 ×106
= 100 ns
can be expected in the transmission of eight Time for 1 byte = 8 × 100 =800 ns
sequential 512-byte packets if the system Time for 1526 byte =1526 × 800 ns
BER is 2x10-4? Time for 1526 byte =1.2208 ms
Notations:
Note for ethernet (original)
BER = Bit Error Rate 1526 byte = 1500 bytes (data) + 8 bytes
(preamble) + 6 bytes (destination address)
Solution:
+ 6 bytes (source address) + 2 bytes (type)
8 𝑝𝑎𝑐𝑘𝑒𝑡𝑠 𝑥 512 𝑏𝑦𝑡𝑒𝑠 = 4,096 𝑏𝑦𝑡𝑒𝑠 + 4 bytes (frame check sequence)
32,768 ∴ The time required for a 1500 byte of
= 3.2768 𝑠𝑒𝑡𝑠 𝑜𝑓 10,000 𝑏𝑖𝑡𝑠
10,000 data to be transmitted is 1.2208 ms.
𝐸𝑟𝑟𝑜𝑟 = 2(3.2768) PROBLEM 4
Calculate the transmitted data stream for the
𝐸𝑟𝑟𝑜𝑟 = 6.5536 𝑏𝑖𝑡𝑠 following data and CRC generating
∴ The errors in the transmission is 7 polynomials: Data G (x) = x raised to the
bits. power of 7 + x raised to the power of 4 + x
raised to the 2nd power + x + one, P (x) = x
PROBLEM 2 raised to power 5 + x raised to power 4 + x
+ one
In an office with 20 PCs, how many
interconnecting wires or links are required so
that any computer can communicate with
Notations:
CRC = Cyclic Redundancy Check
any other computer?
BCS = Block Check Sequence
Notations:

N = number of links
Solution: PROBLEM 6
Multiply G(x) by the number of bites on the For an ethernet bus that is 500 meters in
CRC code. length, using a cable with a velocity factor of
x5 (x7+ x5+ x4+ x2+ x1+ x0) 0.66, and a communication rate of 10 Mb/s,
= x12+ x10+ x9+ x7+ x6+ x5 calculate the total number of bits that would
=10110111000000 be sent by each station before it detects a
Divide the data message G(x) by a generator collision, if both stations begin to transmit at
polynomial function P(x). the same time.
Discard the quotient and the remainder is
truncated to 16 bits. Notations:
Add to the message as the block sequence Vp = propagation velocity (m/s)
(BCS). C = wave velocity (m/s)
The result is the transmitted data stream. Vf = velocity factor
K = total number of bits (bits)
Transmitted Data L = cable length (m)
=1011011101001
∴ The transmitted data stream is Solution:
10011011101001. Vp = c V f
Vp = (300 × 108) (0.66)
PROBLEM 5 Vp = 198 × 108 m/s
A telephone line has a bandwidth of 3.2 kHz K = (10
Mbits
)(
1 sec
)(500m)
sec 198 ×105 𝑚
and a signal-to noise ratio of 36 dB. A signal
K = 25 bits
is transmitted down the line using a three
∴ The total number of bits sent before it
level code. Calculate the maximum
detects a collision is 25 bits.
theoretical data rate taking into account the
presence of noise.
PROBLEM 7
Determine the resolution for an 8-bit linear
Notations:
sign-magnitude PCM for a maximum
C = channel capacity (bit/sec)
decoded voltage of 2.55 Vp.
BW = bandwidth (Hz)
S/N = signal-t-noise ratio (absolute value)
Notations:
DR = dynamic range (absolute value)
Solution:
Vmax = maximum decoded voltage (V)
𝑆
n = number of PCM bits excluding sign bit
C= 3.32BWlog(1+ )
𝑁
C = 3.32(3.2kHz) log (1+3981.07) Solution:
C = 38.25 kbps DR = (
Vmax
)
Resolution
Where: Vmax
𝑆 36 Resolution = DR
𝑁
=log-2(10)=3981.07
2.55Vp
∴ The maximum theoretical data rate is Resolution = 127
= 0.02 V
38.25 kbps. Where:
DR = 2n – 1; n = 8 – 1 (excluding sign bit) PROBLEM 10
DR = 27 – 1 = 127 For a dynamic range of 120, how many bits
∴ The resolution of an 8-bit linear sign are required in the PCM code (including the
magnitude PCM is 0.02V. sign bit)?

PROBLEM 8 Notations:
For a sample rate of 40 kHz, determine the DR = dynamic range (absolute value)
maximum analog input frequency. n = number of PCM bits excluding sign bit
Solution:
Notations: n=
log(DR+1)
log 2
fs = highest frequency to be sampled (Hz) log(120+1)
fa = highest frequency to be sampled (Hz) n= log 2
= 6.9
n ≈ 7 bits (excluding sign bit)
Solution: Total number of bits
fs = 2fa = 40kHz = n + 1 (with 1 sign bit)
where: = 8 bits
fa =
fs ∴ The total number of bits including
2
40 Khz sign bit is 8.
fa =
2
fa = 20 kHz PROBLEM 11
∴ The maximum input frequency is 20
How many Hamming bits would have to be
kHz.
added to a data block containing 128 bits?
PROBLEM 9 Notations:
Determine the dynamic range for a 10-bit
n = number of Hamming bits
sign magnitude code.
m = data bits
Notations: Solution:
DR = dynamic range (absolute value)
2𝑛 ≥ 𝑚 + 𝑛 + 1
n = number of PCM bits excluding sign bit
Try n=7;
Solution: 27 ≥ 128 + 7 + 1
DR = 2n – 1
DR = 29 – 1 = 511 128 ≥ 136 (𝑛𝑜𝑡 𝑡𝑟𝑢𝑒)
Where: Try n=8
n = no. of bits (excluding sign bit)
28 ≥ 128 + 8 + 1
n = 10 – 1 = 9
∴ The dynamic range is 511. 256 ≥ 138 (𝑡𝑟𝑢𝑒)
∴ The number of Hamming bits added is
8.
PROBLEM 12 C = 5000 bps
An asynchronous communication system For a practical channel (with some noise
uses ASCII at 9600 bits/sec with eight bits, present),
one start bit and no parity bit. Express the S
data rate in words per minute. (Assume a C = 3.32BWlog(1 + )
N
word has five characters or letters and one
S
space) 5000 = 3.32(2500) log (1 + )
N
S/N = 3
Solution: In dB,
Bit rate = 9600 bits/sec S/N = 10log 3
Bit rate = 576,000 bits/min S/N = 4.77 dB
10 bits per character: ∴ The signal to noise ratio is 4.77.
8 bits + 1 start bit + one space PROBLEM 14
One word = 5 characters + one space Determine the output voltage of µ-law
The rate in words per minute (wpm) compander with a maximum voltage range of
𝑏𝑖𝑡𝑠 1 𝑤𝑜𝑟𝑑 1 𝑐ℎ𝑎𝑟𝑎𝑐𝑡𝑒𝑟 1 V and an input of 0.25 V.
= (576,000 𝑚𝑖𝑛
) (6 𝑐ℎ𝑎𝑟𝑎𝑐𝑡𝑒𝑟𝑠) ( 10 𝑏𝑖𝑡𝑠 )
Notations:
= 9600 𝑤𝑝𝑚
Vout = output voltage (V)
∴ The data rate in words per minute is
Vin = amplitude of the signal at a particular
9600 wpm.
instant of time (V)
PROBLEM 13
Vmax = maximum uncompressed analog input
For digital communications, determine the amplitude (V)
signal to noise ratio in dB which would be
µ = parameter used to define the amount of
required for an ideal channel with a
compression = 255 (typical)
bandwidth of 2500 Hz.
Solution:
Notations:
𝑉
C = channel capacity (ops) ln( 1 + µ 𝑉 𝑖𝑛 )
𝑚𝑎𝑥
𝑉𝑜𝑢𝑡 = 𝑉𝑚𝑎𝑥
BW = handwidth (Hz) ln(1 + µ)
0.25
N = number of coding levels ln( 1 + (255) 1 )
𝑉𝑜𝑢𝑡 =1
Solution: ln(1 + 255)

For an ideal channel, 𝑉𝑜𝑢𝑡 = 0.75 V

C = 2BWlog 2 N ∴ The output voltage of a µ-law


compander is 0.76 V.
with N = 2(binary coding)
C = 2BW = 2(2500)
PROBLEM 15 PROBLEM 17
For binary phase shift keying (BPSK) What is the Shannon limit for information
modulation with a carrier frequency of 80 capacity for a standard voice band
MHz and an input bit rate of 10 Mbps, communications channel with a signal-to-
determine the minimum Nyquist bandwidth. noise ratio of 1000 (30dB) and a bandwidth
of 2.7 kHz?
Notations:
fn = minimum Nyquist bandwidth (Hz) Notations:
fb = transmission rate (bps) C = channel capacity (bps)
BW = bandwidth (Hz)
S/N = signal-to-noise ratio (absolute value)
Solution:
For BPSK, Solution:
fN = fb = 10 MHz C = BWlog2 (1 + N)
S

∴ The minimum Nyquist bandwidth is


C = 2700log2 (1 + 1000)
10 MHz.
C = 26.9 kbps
∴ The reflector coupling factor is 0.91.
PROBLEM 16
In A-law companding, determine the output
PROBLEM 18
voltage if the input voltage to the
commander is 0.8V and the maximum In PCM, assuming uniform quantization what
possible input voltage is 1V. is the signal-to-noise ratio in dB for an 8-bit
code word?
Notations:
Notations:
Vin = amplitude of the signal at a particular
instant of time S/N= signal-to-noise ratio (dB)
Vmax = maximum uncompressed analog input
N= number bits
amplitude
A = parameter used to define the amount of Solution:
compression = 87.6 (typical) 𝑆
Vout = output voltage (V) 𝑁
= 6𝑛 + 1.8

𝑆
= 6(8) + 1.8
Solution: 𝑁
A𝑉in
1 +ln 𝑆
Vout = Vm
𝑁
= 49.8𝑑𝐵
1+lnA
0.8
1 +ln(87.6)
Vout = 1 ∴ The signal-to-noise ratio is 49.8dB.
1+ln 87.6
Vout = 0.96V PROBLEM 19

∴ The output voltage is 0.96V. What is the signal-to-noise ratio in a PCM


system with 512 quantizing levels using
uniform quantization?
Notations: Notations:

S/N= signal-to-noise ratio (dB) V1 = strongest signal(V)

M=number of output conditions possible V2 = weakest signal(V)

Solution: Solution:
𝑆 3 𝑉
𝑁
= 10log(2 𝑀2 ) Blocking dynamic range = 20log 𝑉1
2

𝑆 3 𝑉1
= 10log[ (512)2 ] 60𝑑𝐵 = 20𝑙𝑜𝑔
𝑁 2 0.6µ𝑉
𝑆 60
𝑁
= 55.95 𝑑𝐵 𝑉1 = 0.6µ𝑉𝑙𝑜𝑔−1 20

∴ The signal-to-noise ratio is 55.95dB. 𝑉1 = 0.6𝑚𝑉 = 600µ𝑉


PROBLEM 20 ∴ The strongest signal is 600µV.
In digital modulation system, what is the PROBLEM 22
input rate to MSK modulator, if the mark
frequency is 5MHz. Assume n is equal to Determine the channel capacity of a 4 kHz
unity channel with S/N = 10dB.

Notations: Notations:

fm = mark frequency (Hz) C= channel capacity (bps)

fs = space frequency (Hz) BW = bandwidth (Hz)

fb = bit rate (bps) S/N = signal-to-noise ratio (absolute value)

Solution: Solution:
𝑆
𝑓𝑚 𝑎𝑛𝑑 𝑓𝑠 = 𝑛( 𝑏 )
𝑓 𝐶 = 3.32𝐵𝑊𝑙𝑜𝑔(1 + 𝑁)
2

2𝑓𝑚 2(5 𝑀𝐻𝑧) 𝐶 = 3.32(4 𝑘𝐻𝑧)log(1 + 10)


𝑓𝑏 = ( 𝑛
) = 1
𝐶 = 13.83 𝑘𝑏𝑝𝑠
𝑓𝑏 = 10 𝑀𝑏𝑝𝑠
Where:
∴ The effective radiated power is
𝑆 𝑆
1577.61 W. 𝑁
(𝑑𝐵) = 10𝑙𝑜𝑔 𝑁

PROBLEM 21 𝑆
= 𝑙𝑜𝑔−1 (
10𝑑𝐵
) = 10
𝑁 10
A receiver has a sensitivity of 0.6 uV and a
∴The channel capacity is 13.82 kbps.
blocking dynamic range of 60dB. What is the
strongest signal that can be present along PROBLEM 23
with a 0.6uV signal without blocking taking
If the transmission rate of a digital
place?
communication system of 10Mbps
modulation scheme used is 16QAM, ∴The data rate of the transponder is 72
determine the bandwidth efficiency. Mbps.

Notations: PROBLEM 25

fb = transmission rate (bps) For digital communications, determine the


signal to noise ratio in dB which would be
fN = minimum bandwidth required (Hz)
required for an ideal channel with a
Solution: bandwidth of 2500 Hz.
𝑓
𝐵𝑊 𝑒𝑓𝑓𝑖𝑐𝑖𝑒𝑛𝑐𝑦 = 𝑓 𝑏 = 𝑓
𝑓𝑏 Notations:
𝑁 𝑏 /4

𝑏𝑖𝑡𝑠
C = channel capacity (bps)
𝐵𝑊 𝑒𝑓𝑓𝑖𝑐𝑖𝑒𝑛𝑐𝑦 = 4 𝑤ℎ𝑒𝑟𝑒 𝑓𝑜𝑟 16𝑄𝐴𝑀,
𝑐𝑦𝑐𝑙𝑒
BW = bandwidth (Hz)
𝑓𝑏
𝑓𝑜 =
4 N = number of coding levels
∴The bandwidth efficiency is 4 S/N = signal-to-noise ratio (dB)
bits/cycle.
Solution:
PROBLEM 24
For an ideal channel,
Assuming there is negligible noise, what
𝐶 = 2𝐵𝑊𝑙𝑜𝑔2 𝑁
would be the data rate of one satellite
transponder used for binary data 𝐶 = 2𝐵𝑊 = 2(2500)
transmission?
𝐶 = 5000 𝑏𝑝𝑠
Notations:
With N = 2 (binary coding)
C = channel capacity (bps)
For a practical channel
BW = bandwidth (Hz)
(with some noise present)
N = number of coding levels
𝑆
𝐶 = 3.32𝐵𝑊 log(1 + )
Solution: 𝑁

Without noise, 𝑆
5000 = 3.32(2500) log(1 + )
𝑁
𝐶 = 2𝐵𝑊𝑙𝑜𝑔2 𝑁)
𝑆/𝑁 = 3
𝐶 = 2(36 𝑀𝐻𝑧)𝑙𝑜𝑔2 2
In dB,
𝐶 = 72 𝑀𝑏𝑝𝑠
𝑆
= 10 log 3 = 4.77 𝑑𝐵
Where: 𝑁

N = 2 (binary coding) ∴The signal-to-noise ratio is 4.77dB.

BW = 36 MHz (typical bandwidth per


transponder)
PROBLEM 26 𝐶 = 50.3 𝑘𝑏𝑝𝑠

A telephone line has a bandwidth of 3.2 kHz Where:


and a signal-to-noise ratio of 34dB. A signal
𝑆 15
is transmitted down this line using a four- = 𝑙𝑜𝑔−1 ( ) = 31.6
𝑁 10
level code. What is the maximum theoretical
data rate? ∴The channel capacity is 50.3 kbps.

Notations: PROBLEM 28

C = channel capacity (bps) An A/D converter uses 14-bit numbers and


has a voltage range of -6 to +6V. What is the
BW= bandwidth (Hz) resolution of digitization expressed as the
S/N = signal-to-noise-ratio (absolute value) smallest voltage increment?

Solution: Notations:

𝑆 DR = dynamic range (absolute value)


𝐶 = 3.32𝐵𝑊𝑙𝑜𝑔(1 + )
𝑁
V = total voltage range (V)
3)
𝐶 = 3.32(3.2𝑥10 log(1 + 2511.87)
n = number of PCM bits excluding sign bit
𝐶 = 36.12 𝑘𝑏𝑝𝑠
Solution:
Where:
The total voltage range is -6V to +6V =12V
𝑆
= 34𝑑𝐵 = 2511.87 𝑉 12𝑉
𝑁 Resolution = =
𝐷𝑅 2𝑛 −1
∴The maximum data rate is 36.12 kbps. 12𝑉
Resolution = 214 −1
PROBLEM 27
Resolution = 732.47 µ𝑉
A radio channel has a bandwidth of 10 kHz
and a signal-to-noise ratio of 15dB. What is ∴ The resolution of digitization
the maximum data rate that can be expressed as the smallest voltage
transmitted using any system? range is 732.47µV.

Notations: PROBLEM 29

C = channel capacity (bps) Calculate the peak power in a 120 millivolts


NRZ square wave on a 600-ohm channel of
BW = bandwidth (Hz)
a digital system.
S/N = signal-to-noise ratio (absolute value)
Notations:
Solution:
Pmax = peak power (W)
𝑆
𝐶 = 𝐵𝑊 log(1 + ) V = signal level (V)
𝑁
𝐶 = 10𝑥103 𝑙𝑜𝑔2 (1 + 31.6) Z= impedance (Ω)
Solution: For QPSK, fN =
𝑓𝑏
2
𝑉2 (120𝑥10−3 )2
𝑃𝑚𝑎𝑥 = 𝑍
= 600
= 24µ𝑊 BW efficiency = 𝑓
𝑓𝑏
= 2 bits/ cycle
𝑏 /2

∴ The peak power is 24µW.


∴ The bandwidth efficiency is 2
PROBLEM 30 bits/cycle.

What is the bandwidth required to transmit PROBLEM 32


at a rate of 10Mbits/sec in the presence of a
Given the pulse-spreading constant equal to
28dB S/N ratio?
10ns/m and the cable length equal to 100
Notation: meters, determine the maximum bit rate in
Mbps for UPNRZ transmission.
C = channel capacity (bps)
Notation:
BW = bandwidth (Hz)
fL = transmission rate (bps)
S/N = signal-to-noise ratio (absolute value)
Δt = pulse spreading (sec/m)

L = cable length (m)


Solution:
Solution:
𝑆
𝐶 = 3.32𝐵𝑊 log(1 + 𝑁)
For UPNRZ transmission,
𝑐 10𝑀𝑏𝑝𝑠
𝐵𝑊 = 𝑆 = 3.32 log(1+630.96) 1
3.32 log(1+ )
𝑁 𝑓𝑏 =
2𝛥𝑡 𝑥 𝐿
𝐵𝑊 = 1.075 𝑀𝐻𝑧
1
𝑓𝑏 = = 0.5 𝑀𝑏𝑝𝑠
Where: 2(10𝑥10−9 )(100)
𝑆 28
= 𝑙𝑜𝑔−1 (10) = 630.96 ∴ The maximum bit rate is 0.5 Mbps.
𝑁

∴ The bandwidth required is 1.075 MHz.

PROBLEM 31

Determine the bandwidth efficiency for QPSK


modulation scheme at a transmission rate of
10Mbps.

Notations:

fb = transmission rate (bps)

fN = minimum bandwidth required (Hz)

Solution:
𝑓
BW efficiency= 𝑓𝐷
𝑁

You might also like